Разное

Смешные и сложные загадки с ответами: Смешные логические загадки с ответами с подвохом и шуточные

Содержание

Смешные загадки для детей — смешные загадки на логику с ответами

Ребенок хочет отдохнуть от школы, а вы боитесь, что он «расслабится» и перестанет учиться? Предложите ему отгадать пару смешных загадок на логику – он получит удовольствие, а заодно «прокачает» пять навыков мышления и научится отвечать на самые каверзные вопросы.

Зачем решать задачи на логику?

Поиск отгадки – своеобразная тренировка для мозга. В процессе размышлений ребенок учится анализировать и оценивать факты, грамотно обрабатывать информацию и мыслить не по шаблону. Найденное решение формирует успех и уверенность в себе, а игровой формат обучения не дает заскучать и отвлечься. Ребенок становится собранным, усидчивым и находчивым, быстрее усваивает новое и начинает по-настоящему радоваться победам.
Шуточные загадки помогают школьнику расслабиться, взрослым – снять стресс и от души посмеяться. Если чувствуете усталость — познакомьтесь с подборкой, которую мы составили специально для вас.

Смешные загадки для детей

Загадка №1.Смешные и сложные загадки с ответами: Смешные логические загадки с ответами с подвохом и шуточные


Задание: Воробей может съесть горсточку зерна, а лошадь не может. Почему?

Показать ответ

Ответ: Воробей слишком маленький, чтобы съесть лошадь.

Загадка №2.
Задание: За что учеников выгоняют из класса?

Показать ответ

Ответ: За дверь.

Загадка №3.
Задание: Какое колесо не крутится при правом развороте?

Показать ответ

Ответ: Запасное.

Загадка №4.
Задание: Какой конь не ест овса?

Показать ответ

Ответ: Шахматный конь.

Загадка №5.
Задание: Как каплю превратить в цаплю?

Показать ответ

Ответ: Заменить букву «к» на «ц».

Отлично! А теперь отодвиньте от экрана детей – мы загадаем загадки только для взрослых.

Смешные загадки для взрослых

Загадка №1.Смешные и сложные загадки с ответами: Смешные логические загадки с ответами с подвохом и шуточные
Задание: Что нужно делать, если вы сели в машину, а ноги до педалей не достают?

Показать ответ

Ответ: Просто пересядьте на водительское кресло.

Загадка №2.
Задание: Какая разница между попом и Волгой?

Показать ответ

Ответ: Поп — батюшка, а Волга — матушка.

Загадка №3. «Не Бенджамин Баттон».
Задание: Я – высокий, когда молодой. Я – маленький, когда старый. Что я такое?

Показать ответ

Ответ: Свечка.

Загадка №4.
Задание: Не лёд, а тает, не лодка, а уплывает.

Показать ответ

Ответ: Зарплата.


Посмеялись? Отлично! Мы уверены, что смех продлевает жизнь. А если вы слишком серьезны, загляните на сайт Умназии —найдете 3484 авторские задачи, и какие-то из них вам обязательно приглянутся. И, конечно, захватите с собой детей — в Умназии они смогут развивать пять навыков мышления, участвовать в олимпиадах, зарабатывать личные достижения и самостоятельно выбирать траекторию обучения.Смешные и сложные загадки с ответами: Смешные логические загадки с ответами с подвохом и шуточные

Приходите в Умназию — у нас не бывает скучно.

Математика и логика для детей 7-13 лет

Развиваем логическое мышление через решение сюжетных математических задач в интерактивном игровом формате

узнать подробнее

Читайте также:


 

Загадки про чувства, явления, объекты — про эхо, тень, любовь, сон и на другие темы

Молчуна перемолчит, крикуна перекричит — самая популярная загадка про эхо. Но не единственная, у нас вы найдёте и другие загадки про эхо. А ещё у нас есть загадки про дорогу, сон, тень, огонь, имя и многие другие. Загадки на любой цвет и вкус, русские народные и новые загадки, для детей и взрослых, длинные и короткие.

Смотрите загадки с ответами: тень, дорога, огонь, эхо, любовь, дым, масленица, печь, имя, сон.

Не было и не будет.
А если будет, весь мир погубит.
Ответ: бессмертие
Ризванова Регина, Намонган
5569

Всю неделю отдыхаем,
Всех блинами угощаем.Смешные и сложные загадки с ответами: Смешные логические загадки с ответами с подвохом и шуточные
Холод зимний провожаем,
А весну с теплом встречаем.
Ответ: Масленица
Миронова Галина, Москва
1899

Прогони от нас метели,
Прокати на карусели.
Растопи холодный лёд,
Пусть весна скорей придёт!
Ответ: Масленица
Миронова Галина, Москва
1508

Этот праздник — объеденье!
Напечем блины с утра.
К ним — сметана и варенье
И, конечно же, икра!

Ответ: Масленица
Миронова Галина, Москва
1388

Был водой, теперь летаю.
С виду я туман напоминаю.
Ответ: пар
5550

Ножницы, шампунь, расческа,
Всем я делаю прически,
Стригу и взрослых, и детей.
Отгадай меня скорей!
Ответ: парикмахер
Котелевская Юлия, Первоуральск
1768

Взрослые лихо снежками кидаются,
Блинами, как дети, они объедаются,
На санках все с горок катаются.
Как же всё это у нас называется?
Ответ: Масленица
2561

Бегать нужно там, скакать,
Веселиться и играть,
Только раз зайдёшь туда:
Начинается игра!
Ответ: Спортивный зал
Анастасия
1742

В землю он весной нырял —
Поле всё перепахал.Смешные и сложные загадки с ответами: Смешные логические загадки с ответами с подвохом и шуточные
Ответ: плуг
1790

Он мост мостил –
Топора не просил.

Столбы забивал,
Гвоздей не брал.
Ночью реки сковал,
Синим льдом покрывал.
Окна в доме расписал,
Ели в иней нарядил
И лисицам и зайчаткам
Шубы старые сменил.
И под Новый год ребятам
Сам подарки разносил.
Ответ: Дед Мороз
716

Первый слог простой предлог.
Слог второй звериный рот.
Если слово ты прочтёшь
Быстро в бездну упадёшь!
Ответ: Пропасть — ПРО + ПАСТЬ
Стрельникова Татьяна, Оренбург
386

Ищет он во тьме маяк, потому что он…
Ответ: Моряк
Ульянова Дарья , Красноярск
465

Где встречается такое, что земля над головою?
Ответ: Туннель
)))))))) Renesmee, L. A
431

Крылья как у птицы,
Круг над головой,
Живёт над нашей крышей,
Хранит он нас с тобой!
Ответ: Ангел
Трубникова Ирина, Ростов-на-Дону
354

Ёлочка с игрушками,
Клоуны с хлопушками,

Веселится весь народ
Что за праздник?
Ответ: Новый Год
Малина Софья, Москва
228

Я скажу ему «машина»,
И ответит он мне «шина».Смешные и сложные загадки с ответами: Смешные логические загадки с ответами с подвохом и шуточные
Ответ: Эхо
Малец Даниэль, Минск
375

Она красива и мила,
А имя ей от слова «зола».
Ответ: Золушка
788

Мы слепили снежный ком,
Шляпу сделали на нём,
Нос приделали — и вмиг
Получился …
Ответ: Снеговик
антоник полина, тобольсск
313

Крыльями машет, а улететь не может.
Ответ: Ветряная мельница
Яковлева Арианна, Краснодар
339

В нём живёт моя родня,
Мне без неё не жить ни дня.
В него стремлюсь всегда и всюду,
К нему дорогу не забуду.
Я без него дышу с трудом,
Мой кров, родимый, тёплый…
Ответ: Дом
440

Друг на дружке ровно в ряд

Эти кубики стоят,
В каждом есть окно и вход,
В каждом кто-нибудь живет.
Ответ: Квартиры
442

Этот южный материк
И не мал, и не велик
Нунатаки, снег и льдины,
И веселые пингвины
Здесь давно нашли приют.Смешные и сложные загадки с ответами: Смешные логические загадки с ответами с подвохом и шуточные
Материк тот как зовут?
Ответ: Антарктида
Вешникова Елена, с. Южно-Александровка Красноярский край
252

Это — средство обращения,
Это — средство накопления.
Средство стоимости также,
Также средство платежа.
Ответ: Деньги
Вешникова Елена, с. Южно-Александровка Красноярского края
174

Ночью ходит,
Днём спит.
Как вечер подходит,
За всеми следит.
Ответ: Вампир
дендеря галина, Омск
282

В космосе сквозь толщу лет
Ледяной летит объект.
Хвост его — полоска света,
И зовут объект …

Ответ: Комета
320

В тулупе с красным кушаком и с замечательным мешком.
Ответ: Дед мороз
якимёнок ангелина
166

Весь день за мной шагала, а в темноте пропала.
Ответ: Тень
парогская диана, нижневартовск
387

Стоят два кола.
На колах бочка,
На бочке кочка,
На кочке лес дремучий,
А из бочки две палки торчат.Смешные и сложные загадки с ответами: Смешные логические загадки с ответами с подвохом и шуточные
Ответ: Человек
Швецова Соня, Москва
227

Что за штука
Есть у турка?
Ответ: Штукатурка
841

Маленькое озеро, а дна не видно.
Ответ: Колодец
Попылькин Кирилл, Ишим
294

Что нельзя увидеть и потрогать, а можно только услышать?
Ответ: Голос
журавлёва настя, таганрог
249

Не пловец, а плывущему плыть помогает.
Не хитрец, а мигает, мигает, мигает.
Ответ: Маяк
844

Зимой заклеено оно,
А летом открывается.
Оно кончается на «О»
И так же начинается.
Ответ: Окно
510

То в стороне стоит молчком,
Задрав носишко, чванится.
То угощает нас чайком,
Не уставая кланяться.
Ответ: Чайник
406

Приходит к людям вновь и вновь
Большое чувство под названием…
Ответ: Любовь
731

Бываю платонической,
Бываю эротической,
Но не хочу быть разделённой.
Ответ: Любовь
534

Я — имя женское, я — чувство,
Мой праздник — в феврале.Смешные и сложные загадки с ответами: Смешные логические загадки с ответами с подвохом и шуточные
Живу в сердцах, живу я в душах,
И в воздухе витаю по весне.
Ответ: Любовь
465

Мой символ прост — пылающее сердце,
Но нужно, чтоб была открыта дверца.
Ответ: Любовь
451

Чему все возрасты покорны?
Ответ: Любовь
670

Ангел на крыльях ко мне прилетел,

Своею стрелою мне сердце задел.
Знаю, что больше не будет мне грустно,
Ко мне пришло настоящее чувство.
Ответ: Любовь
311

По нему идут,
Он и там и тут,
Он в твоей квартире,
В школе, в бане, в тире,
И запомнить так легко:
Он всегда под потолком!
Ответ: Пол
279

Не дом, но и не улица,
Высоко, но не страшно.
Ответ: Балкон
599

Тысячу баранов
Одной хворостинкой сгоняю.
Ответ: Борода и бритва
338

Как только разойдусь —
Всё ем и ем.
Но если наемся —
Исчезну совсем.
Ответ: Огонь
1219

Когда всё видишь, то её не видишь,
А когда ничего не видишь, то её видишь.Смешные и сложные загадки с ответами: Смешные логические загадки с ответами с подвохом и шуточные
Ответ: Темнота
336

Он пропищит вам пару строчек, на языке тире и точек.
Ответ: Радист
253

Белый, а не сахар. Холодный, а не лёд.
Ответ: Иней
478

Везут рогатку колоть мохнатку.
Ответ: Вилы и сено
332

Не птица, а летает.
В лоб попадёт — фонарь зажжёт.
Ответ: Камень
330

Чем больше по ней топчутся, тем она лучше.
Ответ: Тропинка
318


Смотрите загадки на другие темы:


Загадки с подвохом и на логику

Отправился человек в море и попал в шторм. Его отнесло на остров, где не было мужчин, а жили только девушки. Утром проснулся он весь в веревках на каком-то ритуале и узнал, что его хотят убить. И попросил он последнее слово. После того, как он сказал его, девушки смастерили ему лодку, дали еду, воду и отправили домой. Что он сказал?
Ответ: пусть меня убьет самая некрасивая
Островерхая Диана
3294

Слушай, шли два брата, нашли 3 апельсина.Смешные и сложные загадки с ответами: Смешные логические загадки с ответами с подвохом и шуточные Не резали, не пилили, как поровну поделили?
Ответ: Шли с Лушей
Воробьва Катя , Абакан
3578

У бога есть, у царя нет,
У Бориса спереди, а у Глеба позади,
У бабы две, а у девки ни одной!
Ответ: Буква б
Настя), Ростов-на-Дону
5180

В Африке жили негр муж и негр жена. Пришло ей время рожать. Они вызвали скорую, женщина родила в машине. Ребенок был белый, зубы тоже белые. Что не так?
Ответ: С зубами не рождаются
Грецкая Виталия
4527

На дне моря лежит сундук. В нем все есть, кроме одного. Чего в нем нет?
Ответ: Пустоты
Затейщикова Лиза, Москва
4750

Слово из 9 букв с 6 согласными подряд.
Ответ: ВЗБЗДНуть
Кузнецов Павел, Электросталь
4462

Однажды утром мужчина замечает, что в одной из шин его машины совсем нет воздуха. Всё же он садится в машину и едет 150 км к своему клиенту. После визита он возвращается обратно. Хотя он не накачивал шину, он мог передвигаться на своем авто без проблем.Смешные и сложные загадки с ответами: Смешные логические загадки с ответами с подвохом и шуточные Почему он мог ездить на автомобиле?
Ответ: Спущенная шина была на запасном колесе
храмова настя
3000

В каком слове 3 буквы л и три буквы п?
Ответ: Параллелепипед
ильичёва лёля, самара
5049

Он бывает деревянный или жидкий. О чём речь?
Ответ: Стул
Лукьянов Алексей, Ставрополь
4206

На одном острове стоит яблоня с мальчиком, а на другом — больница с бабушкой. Между островами мост. Мальчику надо принести бабушке 2 яблока, но мост выдерживает только одного мальчика и одно яблоко. После того, как мальчик пройдет, мост разрушится. А в воде водятся акулы. Как ему перенести яблоки?
Ответ: Мальчик на мосте будет жонглировать
щетинин кирилл, зеленоград
2993

Вы уронили в кофе кольцо. Как его достать, не намочив руки, если у вас ничего нет и кофе нельзя вылить?
Ответ: Кофе в зёрнах
Мега Машустик, Москва
3742

На попе сидит и в попу глядит. Какая профессия человека?
Ответ: Кучер или ямщик
Огурцов Александр, Москва
4000

Дело было зимой.Смешные и сложные загадки с ответами: Смешные логические загадки с ответами с подвохом и шуточные Потерял братец Иванушка свою сестрицу Аленушку. Говорит ему мать: «Иди лесом, полем, полем, лесом, полем, лесом, полем, полем.» Пошел он лесом, полем, полем, лесом…и видит: перед ним большая река. Как ему через реку перебраться?
Ответ: По льду (дело зимой было)
ася, москва
3154

Что мужчина делает один раз в жизни, а женщина постоянно?
Ответ: 1) Вылазит из-под юбки 2) Женщина всегда дышит грудной клеткой, а мужчина только перед смертью
Heinzelmannchen
3234

Какая нота и продукт называются одиноково?
Ответ: Соль
anonim —
3816

У отца Мэри есть 5 дочерей: Чача, Чичи, Чече, Чочо. Как зовут 5 дочь?
Ответ: Мэри
аноним, Москва
5432

От головы до хвоста 12 м, а от хвоста до головы 0 м. Что это?
Ответ: Год (м — месяцев)
Чемров Кирилл, Воронеж
3846

Гномик жил на 6-ом этаже. Он проезжал на лифте 3 этажа, а остальные 3 шёл пешком по лестнице. Почему?
Ответ: Он был маленький и не доставал до кнопки 6-го этажа
Аноним
4417

Стоит мельница, в ней живёт сторож.Смешные и сложные загадки с ответами: Смешные логические загадки с ответами с подвохом и шуточные У сторожа есть собака . На той мельнице четыре окошка, на четырёх окошках по четыре кошки, у каждой кошки по четыре котёнка, у каждого котёнка по четыре мышонка. Сколько всего ног?
Ответ: Две ноги (у животных лапы)
Пейчева Елизавета, Онега
2454

Стоит тюрьма, рядом больница. Вокруг них рельсы,а на рельсах движется кругом с большой скоростью поезд. Одному мальчику надо попасть к деду в тюрьму, а одной девочке к бабушке в больницу. Как им это сделать, если поезд не останавливается?
Ответ: Мальчику надо бросить девочку под поезд, тогда он попадет в тюрьму, а девочку отведут в больницу.
Махмуд
3559

Тюрьма. Вокруг тюрьмы обводная река. 3 зека планируют побег в разное время, друг о друге не знают. 1-й зек сбегает из тюрьмы, плывёт через реку, неожиданно его съедает акула. Умер 1-й зек. 2-й сбегает, плывёт через реку, вдруг его заметила охрана тюрьмы, быстро подплыла на катерах, оглушили его, вытащили за волосы и застрелила при попытке бегства. Умер и 2-й зек.Смешные и сложные загадки с ответами: Смешные логические загадки с ответами с подвохом и шуточные 3-й зек сбегает. Переплывает через реку, ничего ему не помешало, побежал дальше и исчез. Сбежал 3-й зек. Вопрос: где я тебя обманул в трёх местах? Если отгадаешь все три обмана, угощу пивом.
Ответ: 1) В реке нет акул 2) Зеки лысые 3) Не угощу я тебя
Денис Олегович
3500

В один город пришёл мудрец. Всё он знал на свете. Приходили к мудрецу люди за советом, и всем помогал мудрец. В городе жил парнишка. Прослышав, что в городе появился мудрец, парнишка решил проверить способности этого мудреца. Парнишка был хитёр. Поэтому он поймал бабочку и зажал её между ладонями так, что бы можно было её отпустить или раздавить. И вот пришёл этот парнишка к мудрецу с бабочкой между ладошками.
— Послушай меня! Если ты на самом деле очень мудр и помогаешь людям, отгадай, живая ли бабочка у меня в руке?
Если бы он ответил «живая», парнишка раздавил бы бабочку. Если бы он ответил «мертвая», парнишка выпустил бы бабочку. Что же ответил мудрец?
Ответ: Все в твоих руках
Баженова Виктория, Томск
2439

Четверо ребят играли в футбол во дворе.Смешные и сложные загадки с ответами: Смешные логические загадки с ответами с подвохом и шуточные
Кто-то разбил окно. Хозяйка спросила:
— Кто разбил окно?
Серёжа сказал:
— Окно разбил или Юра, или Миша.
Юра сказал:
— Я окно не разбивал.
Игорь сказал:
— Это сделал Миша.
А Миша сказал:
— Игорь ошибается.
Трое ребят всегда говорят правду, а четвертый неизвестно. Кто разбил окно?
Ответ: Миша
Чемров Кирилл, Воронеж
2271

У директора школы есть брат Николай. Но у Николая нет братьев. Может ли такое быть?
Ответ: Да. Директор школы может быть женщиной
Чемров Кирилл, Воронеж
4084

Ни на одной планете нет. Это есть лишь на Земле.
Ответ: Буквы «З», «Л», «Я»
Чемров Кирилл, Воронеж
3952

У Саши есть отец Иван, у Ивана есть брат Коля, у Коли есть сын Петя, у Пети есть брат. Хотя у Коли только один сын. Как такое может быть?
Ответ: У Пети двоюродный брат Саша
Чемров Кирилл, Воронеж
2526

Два друга считали прохожих. Один сидел на месте и считал всех прохожих.Смешные и сложные загадки с ответами: Смешные логические загадки с ответами с подвохом и шуточные А второй ходил взад-вперёд и считал тех, кто идет ему навстречу. Кто насчитал больше?
Ответ: Одинаково. Второй идёт в одну сторону считает одних в другую других
Чемров Кирилл, Воронеж
1932

Однажды один коллекционер старинных денег увидел в антикварном магазине монету, на которой стояла дата: 175-й год до нашей эры. Эта римская монета была чуть повреждена, но представляла собой большую ценность. Стоила она не дорого. Но коллекционер ее не купил. Почему?
Ответ: Коллекционер понял, что у него в руках подделка, так как мастер, который изготовил монету, не знал, что живет «до нашей эры»
Катя
2454

Моя жизнь может быть измерена в часах. Я служу, когда пожираюсь. Когда я тонкая, я быстра. Когда я толстая, я медленна. Ветер мой враг. Кто я?
Ответ: Свеча
Кирилл, Воронеж
2352

Что теплее шубы?
Ответ: 2 шубы
ВОРОБЬЁВА АЛЁНА, ЧАПАЕВСК
4618

В каком городе течёт кровь?
Ответ: Вена
фадеева евгения, москва
3818

Три человека обедали в ресторане.Смешные и сложные загадки с ответами: Смешные логические загадки с ответами с подвохом и шуточные Обед стоил 25 евро. Каждый достал из кошелька по 10 евро, итого получилось 30. Официант забрал 30 евро и принёс сдачу — 5 евро. 2 евро решили оставить на чай, а оставшиеся 3 евро разделили между собой — каждому по 1 евро. Стали считать: каждый человек потратил на обед 9 евро, но если 3х9 = 27, плюс 2 евро на чай, то получается 29 евро всего. Куда делся ещё 1 евро?
Ответ: Правильно так: 27 (25 обед + 2 на чай) + 3 (сдача) = 30 евро, то есть 2 евро на чай уже входят в 27 евро, а они его неверно отнесли к оставшимся 3 евро
5432

Сколько земли содержит яма диаметром 3 метра и глубиной 3 метра?
Ответ: Нисколько (ямы пустые)
13241

Какими нотами можно измерить расстояние?
Ответ: Ми-ля-ми
15635

Какие созвездия носят названия птиц?
Ответ: Лебедь, Орёл
14831

Какая птица носит название части музыкального инструмента?
Ответ: Гриф
11230

Какая птица носит название корабля?
Ответ: Фрегат
12792

Какая птица носит название каши?
Ответ: Овсянка
12807

Какая птица носит название фрукта?
Ответ: Киви
аноним
3375

В поле лежит мертвый человек.Смешные и сложные загадки с ответами: Смешные логические загадки с ответами с подвохом и шуточные На много миль вокруг ни одной живой души. Рядом с человеком лежит мешок. Кто он и как этот бедняга умер?
Ответ: Это парашютист и парашют у него не открылся
7744

Эту загадку ученик 1-ого класса решает за 5 минут, старшеклассник за 15 минут, студент за 1 час, профессор никогда не решит. Загадка: расшифруйте одтчпшсвдд
Ответ: один два три четыре…
Анисимова Татьяна
3201

Летела стая, совсем не большая. Сколько птиц и каких?
Ответ: Семь сов (~ совсем)
13872

На небе одна, на земле ёе нет, у бабушки две, а у дедушки нет.
Ответ: Буква Б
БАШКАЕВА АЛИСА, ГРОЗНЫЙ
2961

Что видно, если ничего не видно?
Ответ: Пар
Лапина Алиночка, Поспеловка
2652

В лесу она не водится,
В реке она одна,
В сарай не помещается,
А в кошельке их два!
Ответ: Буква К
МАГОМАДОВА АЛИСА, ГРОЗНЫЙ
2529

Жили в лесу три друга: Глухой, Немой, и Слепой. Всё было хорошо. Но как-то Глухой умер.Смешные и сложные загадки с ответами: Смешные логические загадки с ответами с подвохом и шуточные Как теперь Немой скажет Слепому, что их Глухой друг умер?
Ответ: Немому не нужно объяснять, что Глухой умер. Слепой его не видел и не разговаривал с ним
Эфи
2240

Шли по обрыву парень и девушка.
Она: ты меня любишь?
Он: да!
Она: а ты смог бы ради меня сброситься вниз?
Какие два слова сказал он, если они пошли дальше целые и невредимые?
Ответ: Толкни меня
Никитина Анастасия , Челябинск
2413

Что находится в центре Земли?
Ответ: Буква М
Блабанова Кира , Волжский
2879

Два молодых казака, оба лихие наездники, часто бились между собою об заклад, кто кого перегонит. Не раз то один, то другой был победителем. Наконец, это им надоело. Григорий сказал: «Давай спорить наоборот. Пусть заклад достанется тому, чей конь придет в назначенное место вторым, а не первым.» «Ладно!» — ответил Михаил. Казаки выехали на своих конях в степь. Зрителей собралось множество: всем хотелось посмотреть на такую диковинку. Один старый казак стал считать, хлопая в ладони: «Раз! Два! Три!..» Спорщики, конечно, не сместа. Зрители стали смеяться, судить да рядить и порешили, что такой спор невозможен и что спорщики простоят на месте, как говорится, до скончания века. Тут к толпе подошел седой старик, видавший на своем веку разные виды: «В чем дело?» Ему сказали. Ответил старик: «Эге ж! Вот я им сейчас скажу такое слово, что поскачут как ошпаренные.» И действительно, подошел старик к казакам, сказал им что-то, а через полминуты казаки уже неслись по степи во всю прыть, стараясь непременно обогнать друг друга. Но заклад все же выиграл тот, чья лошадь пришла второй. Что сказал старик?
Ответ: Старик шепнул казакам: пересядьте каждый на лошадь противника
Адианова Таня
1542

На острове держали заключённого. Его держали в бетонной камере, где нет дверей и окон, а только маленькая щель, в которую ложили хлеб сухой. Через несколько недель в камере оказалась гора костей. Откуда она взялась?
Ответ: Ему давали хлеб с ухой
Роберт
1631

Если загадок с подвохом вам оказалось мало, посмотрите загадки схожей тематики: прикольные и смешные загадки.
На тему загадок с подвохом для вас может быть интересной наша статья «Как нарисовать 1000 человечков за одну минуту?»



Загадки. Прикольные и смешные загадки с подвохом:



Размер

Толщина

Фон

 Скрыть 
 ответы 
 +шрифт 
 —шрифт 
 +жирн 
 —жирн 
White Cyan LGreen GYellw DpSkBl Coral DPink1 DPink2 SkBlue Orange OlivD1 OlivD2 LBlue PGreen Yellow Gold Blue Green Wheat Chocol Salmon Red HPink DPink Pink VioRed Magent Violet Plum Purple OrRed Bisque Bisqu2 LemC1 LemC2 Corns2 Honey2 Turqu1 Turqu2 SGrn1 SGrn2 Orchi1 Orang1 Gray golrod




Что такое диетическое яйцо?


Ответ

Это яйцо, снесенное курицей, находящейся на диете.



О чем думал Мюнхгаузен, когда летел на ядре?


Ответ

Ничего себе в кегельбан поиграл.



Кто получит бесплатный сыр в мышеловке?


Ответ

Вторая мышка.



Что такое глобус?


Ответ

Это чучело земли.



Какая самая не любимая фраза сапера?


Ответ

Одна нога здесь, другая – там.



Что такое МАРС?


Ответ

Это собачий холод и толстый-толстый слой космической пыли.



Что такое чистоплотность?


Ответ

Это чистомасса на чистообъем.



Куда каждый раз падает ядерная бомба?


Ответ

В эпицентр.



Кто такой истинный патриот своей страны?


Ответ

Это тот, кто больше нигде не был.



Как вы думаете, что это такое : черно-белое на 16 колесах.


Ответ

Это зебра на роликовых коньках.




Поделитесь с друзьями:


Как вы думаете, в чем разница между футболистом и уличным пешеходом?


Ответ

На красный пешеход стоит, а футболист уходит.



Если работа не волк, то что тогда?


Ответ

Произведение силы на расстояние.



Что такое – не пойман, не вор?


Ответ

Депутатская неприкосновенность.



Кто такие вертолёты?


Ответ

Это души погибших танков.



Как называется бумеранг, который не вернулся?


Ответ

Палка.



Если оптимист изобретает колесо, то что изобретает пессимист?


Ответ

Запасное колесо.



Какие зубы у человека появляются последними?


Ответ

Искусственные.



Если во время соревнований сходит лавина, то что происходит с лыжниками?


Ответ

Все финишируют одновременно.



Кто в армии служил, тот в цирке …?


Ответ

не смеется.



Закончите сказку : «3 дня и 3 ночи скакал Иван Царевич, пока …»?


Ответ

пока скакалку не отобрали.




Поделитесь с друзьями:


Загадки с подвохом

Загадки с подвохом — это загадки на внимание, сообразительность и чувство юмора. Их нужно задавать быстро, не давая много времени на раздумья.

 

Готовые сценарии для проведения квеста. Подробную информацию можно посмотреть, нажав на интересующую картинку.

 

Весёлые загадки на смекалку с ответами

На любом празднике удачно подобранные загадки будут хорошим развлечением для гостей и обязательно поднимут настроение всем без исключения! С помощью данных загадок вы можете:

  • Использовать их для заполнения пауз между играми, конкурсами и танцами.
  • Устроить весёлую застольную викторину. Для того, чтобы заинтересовать гостей и облегчить процесс определения победителей, можно купить побольше денег от «банка приколов» и вручать их за правильные ответы. Присутствующие мгновенно включаются в игру и будут стараться заработать много «денежек». Вместо «денег» можно приготовить узкие цветные полоски бумаги и, например, нарисовать на них восклицательный знак – за правильный ответ. В конце викторины желательно наградить сувенирами (или шуточными медалями) трёх самых смекалистых игроков, т.е. тех, у кого оказалось больше всего «денег» или цветных бумажек.
  • Устроить командное соревнование (особенно оно подойдет для подростков): разбить игроков на несколько команд, каждой команде выдать листы с одинаковыми загадками и ручки. Чья команда даст больше верных ответов за отведенное ведущим время, та и будет победителем.

Сколько на берёзе яблок, если у нее восемь веток, на каждой из них по 5 яблок?

Показать

(На берёзе не растут яблоки)

[свернуть]

Где находятся города без домов, реки без воды, а леса без деревьев?

Показать

(На географической карте)

[свернуть]

Несла старушка на базар 50 яиц, а дно упало. Сколько яиц осталось? Ведущий произносит «а дно» как «одно»

Показать

(Все разбились, так как упало дно)

[свернуть]

Сидели на дереве шесть глухарей. Пришел охотник и убил одного глухаря. Сколько птиц осталось?

Показать

(Ни одной — остальные улетели)

[свернуть]

Как спрыгнуть с двадцатиметровой лестницы и не разбиться?

Показать

(Спрыгнуть с первой ступеньки, или, для смелых и ловких, — со второй или третьей)

[свернуть]

Шёл Иван в Москву, зашёл на мельницу. Там 4 окошка, на каждом окошке по 4 кошки. У каждой кошки по 4 котенка, а у каждого котёнка по 4 мышки. Сколько всего ног?

Показать

(Две ноги — у Ивана, остальные — лапы)

[свернуть]

Какой рукой лучше размешивать чай?

Показать

(Чай лучше размешивать ложкой)

[свернуть]

Сколько горошин может войти в один стакан?

Показать

(Нисколько – горошины не ходят)

[свернуть]

Перед кем все люди без исключения снимают шляпы?

Показать

(Перед парикмахером)

[свернуть]

У кого есть шапка без головы и нога без сапога?

Показать

(У гриба)

[свернуть]

За что обычно учеников выгоняют из класса?

Показать

(За дверь)

[свернуть]

Что за обедом нужнее всего?

Показать

(Рот)

[свернуть]

Что нельзя съесть на завтрак?

Показать

(Ужин и обед)

[свернуть]

Какими нотами можно измерить пространство?

Показать

(Ми – ля – ми)

[свернуть]

Что можно приготовить, но нельзя съесть?

Показать

(Уроки)

[свернуть]

Что делает сторож, когда у него на шапке сидит воробей?

Показать

(Спит)

[свернуть]

Кто под проливным дождем не намочит волосы?

Показать

(Лысый)

[свернуть]

Что может быть больше слона и одновременно невесомым?

Показать

(Тень слона)

[свернуть]

На какое дерево садится ворона во время дождя?

Показать

(На мокрое)

[свернуть]

Что станет больше, если его поставить вверх ногами?

Показать

(Число 6)

[свернуть]

Каких камней в море не встретишь?

Показать

(Сухих)

[свернуть]

Что станет с красным шёлковым платком, если опустить его на 5 минут на дно моря?

Показать

(Будет мокрым)

[свернуть]

Как сорвать ветку, чтоб не вспугнуть птицу?

Показать

(Надо подождать, когда она улетит)

[свернуть]

Из какой посуды нельзя ничего поесть?

Показать

(Из пустой)

[свернуть]

Чем заканчивается день и ночь?

Показать

(Мягким знаком)

[свернуть]

Когда человек бывает в комнате без головы?

Показать

(Когда высовывает её из окна)

[свернуть]

Может ли страус назвать себя птицей?

Показать

(Нет, страус не умеет говорить)

[свернуть]

Что у человека под ногами, когда он идет по мосту?

Показать

(Подошва обуви)

[свернуть]

Какой узел нельзя развязать?

Показать

(Железнодорожный)

[свернуть]

Можно ли в решете воды принести?

Показать

(Можно, если она там замерзнет)

[свернуть]

Сколько месяцев в году имеют 28 дней?

Показать

(Все)

[свернуть]

Какая разница между слоном и блохой?

Показать

(У слона могут быть блохи, а у блохи не может быть слонов)

[свернуть]

За чем вода в бутылке?

Показать

(За стеклом)

[свернуть]

Что может путешествовать по миру, оставаясь в одном и том же углу?

Показать

(Почтовая марка)

[свернуть]

Что все люди на земле делают одновременно?

Показать

(Становятся старше)

[свернуть]

Серёжки для простаков?

Показать

(Лапша)

[свернуть]

Когда козе исполнится семь лет, что будет дальше?

Показать

(Пойдёт восьмой год)

[свернуть]

Каким гребнем нельзя расчесывать голову?

Показать

(Петушиным)

[свернуть]

Когда мальчика называют женским именем?

Показать

(Когда он долго спит – Соня)

[свернуть]

Как написать слово «мышеловка» пятью буквами?

Показать

(Кошка)

[свернуть]

Что идет, оставаясь на месте?

Показать

(Дорога)

[свернуть]

Когда человек может мчаться со скоростью гоночного автомобиля?

Показать

(Когда он сидит в нем)

[свернуть]

Какие часы показывают верное время только два раза в сутки?

Показать

(Те, которые остановились)

[свернуть]

Что можно увидеть с закрытыми глазами?

Показать

(Сон)

[свернуть]

Рекомендуем посмотреть:

За чем мы едим?

Показать

(За столом)

[свернуть]

Что легче: килограмм ваты или килограмм железа?

Показать

(Одинаково — по килограмму)

[свернуть]

Какой конь не ест овса?

Показать

(Шахматный, спортивный)

[свернуть]

Какое колесо не крутится при правом развороте?

Показать

(Запасное)

[свернуть]

По чему, когда захочешь спать, идешь на кровать?

Показать

(По полу)

[свернуть]

Какое животное может легко поменять голову?

Показать

(Вошь)

[свернуть]

Когда черной кошке легче всего пробраться в дом?

Показать

(Когда дверь открыта)

[свернуть]

На окне сидит, по-французски говорит?

Показать

(Француз)

[свернуть]

Какая разница между молнией и электричеством?

Показать

(За молнию не надо платить)

[свернуть]

Какой возраст у ребёнка самый хороший?

Показать

(Когда вы уже не водите его за руку, а он ещё не водит вас за нос)

[свернуть]

Что теплее шубы?

Показать

(Две шубы)

[свернуть]

Две спины, одна голова, шесть ног. Что это такое?

Показать

(Человек на стуле)

[свернуть]

Что это: две руки, два крыла, два хвоста, три головы, три туловища и восемь ног?

Показать

(Всадник, держащий в руках курицу)

[свернуть]

Маленькая, сморщенная, есть в каждой женщине?

Показать

(Изюминка)

[свернуть]

Назовите слово, в котором 40 гласных

Показать

(Сорока — сорок «А»)

[свернуть]

Рта нет, языка нет, а говорит о каждом

Показать

(Зеркало)

[свернуть]

На какой вопрос нельзя ответить «да»?

Показать

(Вы спите?)

[свернуть]

На какой вопрос нельзя ответить «нет»?

Показать

(Вы живы?)

[свернуть]

В каком случае шесть детей, две собаки, четверо взрослых, забравшись под один зонтик, не намокнут?

Показать

(Если не будет дождя)

[свернуть]

Что есть только у слонов, и ни у каких других животных больше нет?

Показать

(Слонята)

[свернуть]

Что такое 90-60-90?

Показать

(Езда мимо гаишника)

[свернуть]

Маленький, желтенький, в земле ковыряется

Показать

(Вьетнамец мину ищет)

[свернуть]

Маленький, желтенький, в небе кувыркается

Показать

(Вьетнамец мину нашел)

[свернуть]

Два гвоздя упали в воду. Как фамилия грузина?

Показать

(Заржавели)

[свернуть]

Что такое: самое доброе в мире привидение с моторчиком?

Показать

(Запорожец)

[свернуть]

В небе одна, в земле нету, а у бабы их целых две

Показать

(Буква «Б»)

[свернуть]

Что такое: зелёное, лысое и скачет?

Показать

(Солдат на дискотеке)

[свернуть]

Шёл охотник мимо башни с часами. Достал ружьё и выстрелил. Куда он попал?

Показать

(В полицию)

[свернуть]

Без окон, без дверей, а внутри сидит еврей? Что это?

Показать

(Сара беременна)

[свернуть]

Что это такое: на потолке сидит, лампочку грызет?

Показать

(Потолковый лампогрыз)

[свернуть]

Маленький, серенький, на слона похож

Показать

(Слоненок)

[свернуть]

Чем их больше, тем вес меньше. Что это?

Показать

(Дырки)

[свернуть]

Что принадлежит вам, однако другие этим пользуются чаще, чем вы сами?

Показать

(Имя)

[свернуть]

У человека — одна, у вороны — две, у медведя — ни одной. Что это?

Показать

(Буква «О»)

[свернуть]

Сколько яиц можно съесть натощак?

Показать

(Одно)

[свернуть]

Сколько минут нужно варить крутое яйцо?

Показать

(Нисколько — оно уже сварено)

[свернуть]

Что нужно сделать, если вы сели в машину, а ноги до педалей не достают?

Показать

(Пересесть на водительское сиденье)

[свернуть]

Растение, которое всё знает.

Показать

(Хрен)

[свернуть]

Как называется тень от растения хрен?

Показать

(Хренотень)

[свернуть]

Воробей может съесть горсточку зерна, а лошадь не может. Почему?

Показать

(Воробей слишком маленький, чтобы съесть лошадь)

[свернуть]

Светит, но не греет.

Показать

(15 лет строгого режима)

[свернуть]

Где ещё кроме ёлки дети кричат «Дед Мороз, выходи!»?

Показать

(Возле туалета)

[свернуть]

Почему у козы глаза грустные?

Показать

(Потому что муж козёл)

[свернуть]

Как поймать тигра в клетку?

Показать

(Тигра в клетку не бывает, тигр в полоску)

[свернуть]

Два слова, разбивающие все доказательства?

Показать

(И чё?)

[свернуть]

Когда же ребёнок станет конем?

Показать

(Никогда — он ребёнок)

[свернуть]

Как называют корову, которая не даёт молока?

Показать

(Жадина-говядина)

[свернуть]

Кто ещё кроме вампира не отображается в зеркале и не отбрасывает тень?

Показать

(Профессиональный фотограф)

[свернуть]

Самый дорогой кофе в мире?

Показать

(Кофе, пролитый на клавиатуру ноутбука)

[свернуть]

Рекомендуем посмотреть:

 

Прикольные загадки для уже разгулявшейся и не очень трезвой компании взрослых:

То висячий, то стоячий, то холодный, то горячий

Волосатая головка за щеку летает ловко

Показать

Зубная щётка

[свернуть]

Без рук, без ног, на бабу скок!

Показать

Коромысло

[свернуть]

Беру двумя руками, сую между ногами, пять минут потею, а потом балдею

Показать

Велотренажер

[свернуть]

В тёмной комнате, на белой простыне — два часа удовольствия

Показать

Киносеанс

[свернуть]

Волос на волос, тело на тело, и получается тёмное дело

Показать

Глаза

[свернуть]

Туда, сюда, обратно, тебе и мне приятно

Показать

Качели

[свернуть]

Красная головка, работает ловко

Показать

Дятел

[свернуть]

Между ног болтается, на «Х» начинается?

Показать

Хвост

[свернуть]

Мы ребята удалые — ищем щели половые

Показать

Тараканы

[свернуть]

Тихо сзади подошёл, дважды всунул и пошёл

Показать

Тапочки

[свернуть]

Стоит баба на полу, приоткрыв свою дыру

Показать

Печка

[свернуть]

Ты помни его немножко — станет твёрдый, как картошка

Показать

Снежок

[свернуть]

У какого молодца утром капает с конца?

Показать

Самовар или Кран

[свернуть]

Мальчик с девочкой в траве что-то делали на «Е»

Показать

ели землянику

[свернуть]

Чтобы спереди погладить, надо сзади полизать

Показать

Почтовая марка

[свернуть]

Что такое: входит сухим, выходит влажным, дарит тепло и радость?

Показать

Пакетик с чаем

[свернуть]

Назовите слово из 5 букв, содержащее буквы: п, з, д, а.

Показать

Запад

[свернуть]

Выше колена, пониже пупка, дырка такая, что влезет рука

Показать

Карман

[свернуть]

Что такое синее золото?

Показать

Любимая жена напилась

[свернуть]

Он от бабушки ушёл, и от дедушки ушёл… Что это?

Показать

Секс

[свернуть]

Почему у Кощея Бессмертного не было детей?

Показать

Потому что у него всего одно яйцо, и то за тридевять земель

[свернуть]

Из трех букв состоит,

На «Х» начинается,

Когда работает — стоит,

Когда кончает — кланяется.

Что такое: у мужчины на 3 буквы, у женщины на 5 букв?

Показать

Чуб, чёлка

[свернуть]

Что удлиняется, когда его берут в руки, пропускают между грудей и засовывают в отверстие?

Показать

Ремень безопасности

[свернуть]

Также вас могут заинтересовать:

  • Квест для мужчины: для мужа, для любимого человека, для друга на День рождения или Юбилей
  • Квест для женщины: для жены, для любимой девушки, для подруги, сестры, дочки или сотрудницы на День рождения (Юбилей)
  • Командная квест-игра в помещении (в квартире, коттедже, офисе), или поиск спрятанного сюрприза по подсказкам
  • Квест-игра, или поиск спрятанных подарков на День рождения ребёнка 6, 7, 8, 9, 10 лет
  • Командная квест-игра для детей
  • Новогодняя квест-игра: поиск спрятанных подарков в помещении (в квартире, коттедже, офисе)

Смешные логические загадки с ответами с подвохом и шуточные

Загадки для взрослых — с подвохом: лучшая подборка

Загадки с подвохом прекрасный способ повеселиться. Правда помните, что подобный тип загадок для взрослых часто имеет слегка пошлый уклон, а потому выбирать их для развлечения нужно только в том случае, если планируется встреча близких друзей.


Загадки с подвохом для взрослых с ответами

Загадки для взрослых — с подвохом:

  1. Самый известный полупроводник. (Ответ — Сусанин.)
  2. Что нужно сделать, если вы сели в машину, а ноги до педалей не достают? (Ответ — можно попробовать пересесть на водительское место.)
  3. А что нужно сделать, если вы сели на водительское кресло, а ноги до педалей по-прежнему не достают? (Ответ — надо для начала повернуться лицом к рулю.)
  4. Что нужно делать, когда увидишь зеленого человечка? (Ответ — быстро переходить улицу, пока горит зеленый цвет светофора.)
  5. Может ли страус назвать себя птицей? (Ответ — мог бы, если бы умел разговаривать.)
  6. Вы сидите в самолете, перед вами находится лошадь, сзади автомобиль. Где вы находитесь? (Ответ — в парке отдыха, на каруселях.)
  7. В каком случае шесть детей, две собаки, четверо взрослых, забравшись под один зонтик, не намокнут? (Ответ — все очень просто, если на улице не будет дождика.
  8. Имя первой женщины в мире, идеально освоившей летательный аппарат. (Ответ — Баба Яга.)
  9. Эти три телезвезды хорошо известны каждому из нас. Блондина зовут Степан, шатена зовут Филипп. А как зовут лысого? (Ответ — всем известный Хрюша.)
  10. В корзине имеется 5 грибов. Как разделить грибы между пятью грибниками, чтобы каждому досталось поровну, и один гриб остался бы в корзине? (Ответ — 4 грибника получат по одному грибу, а пятый получит корзину с одним грибом.)
  11. Два мальчика играли в шашки. Каждый из них сыграл пять партий, и каждый выиграл такое же количество игр. Как такое могло произойти? (Ответ — просто они играли с разными противниками.)

Загадки про взрослых с ответами

Далее предлагаем вашему вниманию подборку загадок про взрослых людей, и связанные с ними жизненными ситуациями. Ответы, конечно, прилагаются.

Загадки про взрослых с ответами:

  1. Что делает сторож, когда у него на голове сидит воробей? (Ответ — сладко спит.)
  2. После тушения пожара, в лесу был найден труп аквалангиста. До ближайшего водоема более 50 км. Как он там оказался? (Ответ — аквалангист искал рыбок в водоеме, пожарный вертолет его случайно зачерпнул и сбросил на очаг пожара в лесу.)
  3. В пустыне лежит мертвый мужчина. За плечами мешок, на поясе фляга с водой. На многие километры вокруг нет ни единой живой души. От чего умер человек и что в его мешке? (Ответ — человек умер от удара об землю, а в мешке — парашют, который не раскрылся.)
  4. Кто под проливным дождем не намочит волосы? (Ответ — человек с зонтом, или лысый человек.)
  5. Когда человек бывает в комнате без головы? (Ответ — когда высовывает ее из окна или дверей.)
  6. Советским дворникам взяли и укоротили метлы. Зачем? (Ответ — советские дворники любили постоять, опираясь на метлу.)
  7. Косоглазый, маленький, в белой шубке, в валенках? (Ответ — Дед Мороз родом из Чукотки.)
  8. Мальчик упал с 4 ступенек и сломал ногу. Сколько ног сломает мальчик, если упадет с 40 ступенек? (Ответ — всего одну, вторая у него уже сломана.)
  9. Что такое, висит на стене и плачет? (Ответ — альпинист.)
  10. Может ли женщина сделать мужчину миллионером? (Ответ — может, если он миллиардер.)

Загадки для взрослых — веселые


Загадка с ответом

Если ваша цель веселье, то подборка веселых загадок для взрослых поможет сделать ваше времяпровождение максимально легким и непринужденным.

Загадки для взрослых — веселые:

  1. Почему загадки опасны? (Ответ — потому что люди ломают над ними голову.)
  2. Что такое бизнес? (Ответ — это способ отбирать деньги без помощи грубой силы.)
  3. Что будет, если скрестить клопов со светлячками? (Ответ — густая сеть электрификации по всей стране.)
  4. Что такое — 90/60/90? (Ответ — скорость при гаишнике.)
  5. Почему у гориллы большие ноздри? (Ответ — потому что у нее толстые пальцы.)

Шуточные загадки настроение каждого праздника

Удачно подобранные конкурсы и головоломки поднимают настроение на любом торжестве. Оригинальные, сложные, прикольные и смешные загадки станут отменным развлечением для каждого гостя, не оставив равнодушным никого. Они же помогают выполнить ряд следующих задач:

  • заполнение паузы между праздничными играми, танцами и смешными конкурсными программами. Продолжительность занимательного мероприятия незначительная, однако, ее достаточно для расслабления и эмоциональной разрядки гостей;
  • создание веселой праздничной викторины. Для большей заинтересованности гостей можно запастись деньгами от «банка приколов», которые станут награждением за правильное решение. Чем больше человек даст правильных отгадок на смешные загадки, тем больше насобирает денег. Ну а на накопленные средства можно будет «купить» любое желание, предварительно заготовленное ведущим программы. Это вовсе не обязательно будет материальный приз. Как вариант, можно пожелать танец тамады, к примеру, на столе. Или еще что-нибудь в таком роде. Гости моментально подключаться и буквально замельтешат ответами с надеждой на победу;
  • не менее интригующим развлекательным вариантом станут прикольные загадки и для командного соревнования. Для решения задачи потребуется две команды игроков, которые получат шуточные загадки в виде списка на листочке. Победит команда, давшая большее число правильных ответов.

Когда праздник будет в самом разгаре, а гости веселые, время использовать усложненные загадки шутки, отгадывание которых одарит торжество особой располагающей атмосферой. Таким образом, каждый проявит логику и эрудицию, блеснув так называемыми «познаниями» окружающей среды.

Отгадывание смешных загадок: хитрые ходы

Чтобы быстро и правильно решать смешные головоломки и отгадывать загадки на логику, не прибегая к помощи знакомых, друзей и родных, сделаем несколько простых тренировочных ходов. Сподручным материалом станут незатейливые головоломки, отгадки на которые уже знакомы. Вооружившись наглядным «пособием», приступаем к практике.

  1. Руководствуясь ответом, разбираемся в принципе составления. Частым обманным маневром считается вопрос человеку о том, что, согласно его мнения, ему не знакомо. Вопрошаемый приходит в замешательство, тем самым отдаляясь от находящегося на поверхности ответа.
  2. Следующим шагом будет разбор загадки на две главные части. Каждую из них нужно поэтапно осмыслить и попытаться дать ответ. Довольно распространенным считается вопрос о личности, у которой две ноги или три. Понятное дело, что смешная загадка о трех ногах с приколом, поэтому задействуем логическое мышление. Не исключено, что здесь предполагаются пошлые отгадки.
  3. Далее внимательно смотрим на действие в загадке. Если неизвестный предмет, допустим, ходит, значит этим глаголом и руководствуемся в ответе. Не факт, конечно, что ходит он естественным образом, возможно, кто-то заставляет его это сделать.
  4. После выявления второстепенной информации обозначаем время события. Логично действия, начинающиеся с утра и заканчивающиеся вечером, заставляют предмет ходить. Здесь применяется переносное значение. Если же центром времени обозначается день, то возможно применение цифры «12». Одним словом, мыслите с широким кругозором.
  5. Заключительным этапом изучения считается сопоставление полученных отгадок на первые вопросы. Определенно веселые головоломки требуют нестандартной логики, поэтому в решении подразумеваются короткие отгадки рифмованной формы или же двухзначные варианты нестандартного плана.

Влиятельным фактором при разгадывании считается определение вида смешные загадки. Так, незамысловатые головоломки подразумевают стандартный вопрос, а вот смешные загадки, требующие применения креативных математических способностей, часто содержат хитроумный ответ.

Обратите внимание! Ассоциации, логичность которых считается само собой разумеющейся, приемлемой, способны сбить человека с толку. А правильный ответ на смешные загадки, «лежащий» на поверхности, плавно исчезнет с поля зрения. Поэтому, вооружившись хитрым подходом, не теряйте из вида даже самые незначительные, на первый взгляд, мелочи, ведь именно они «сигналят» и направляют на нужное решение.

Загадки для взрослых — логические


Логические загадки для взрослых

Если среди ваших друзей и близких есть люди, любящие не совсем банальные загадки, то следующая подборка вам точно понадобится.

Загадки для взрослых — логические:

  1. Как далеко в глубь леса может забежать заяц? (Ответ — до середины леса, так как далее он уже будет выбираться из леса.)
  2. Сколько программистов нужно, чтобы закрутить лампочку? (Ответ — ни одного. Это аппаратная проблема, программисты их не решают.)
  3. Какое колесо не крутится при правом развороте? (Ответ — запасное.)
  4. По чему, когда захочешь спать, идешь на кровать? (Ответ — по полу.)
  5. Что надо сделать, чтобы четыре парня остались в одном сапоге? (Ответ — снять с каждого по сапогу.)
  6. Каких камней в море нет? (Ответ — сухих.)
  7. На сучке сидит ворона. Что надо сделать, чтобы отпилить сук, не потревожив ворону? (Ответ — дождаться пока она улетит.)
  8. На каком языке говорят молча? (Ответ — на языке жестов.)
  9. Вы опередили лыжника, который находился на второй позиции. Какое место вы теперь занимаете? (Ответ — второе, вы заняли его место.)
  10. У отца Мэри есть 5 дочерей: Николь, Агата, Эжени, Лаура. Как зовут 5 дочь? (Ответ — девочку зовут Мери.)
  11. Электричка едет на восток со скоростью 80 км/час. В какую сторону летит дым во время движения? (Ответ — у электрички не может быть дыма. Она передвигается благодаря электричеству.)
  12. Вы вошли в темную комнату взяв коробок спичек, на столе стоит свечка, керосиновая лампа и газовая плита. Что вы зажжете первым? (Ответ — чтобы стало светло, придется зажечь спичку.)
  13. Есть ли где такое поле, где конь через коня прыгает? (Ответ — шахматное.)
  14. Ваши друзья и знакомые используют это чаще чем вы, но это принадлежит вам.
    О чем идет речь? (Ответ — ваше имя.)
  15. Шла женщина в Москву, а навстречу – три мужика. У каждого по мешку, в каждом мешке по кошке. Сколько существ направлялось в Москву? (Ответ — только женщина шла в Москву. Мужчины уже возвращались обратно.)

Загадки для взрослых — прикольные


Прикольные загадки для взрослых

Ну и еще немного загадок для веселой и шумной компании.

Загадки для взрослых — прикольные:

  1. Что такое — падают с ветки золотые монетки? (Ответ — обычное явление в стране дураков.)
  2. Что такое — ходит по стене и играет? (Ответ — муха с плеером в ушах.)
  3. Идет ежик лысый — сколько ему лет? (Ответ — 18, его в армию забирают.)
  4. Чем отличается женское общежитие от мужского? (Ответ — в женском общежитии посуду моют после еды, а в мужском – перед.)
  5. Бывает маленьким, большим. Железо очень дружит с ним. С ним и незрячий непременно найдет иголку в стоге сена. (Ответ — магнит.)
  6. Что такое диетическое яйцо? (Ответ — это яйцо, снесенное курицей, находящейся на диете.)
  7. О чем думал Мюнхгаузен, когда летел на ядре? (Ответ — вот это в боулинге поиграл.)
  8. Кто получит бесплатный сыр в мышеловке? (Ответ — вторая мышка.)
  9. Какая самая не любимая фраза сапера? (Ответ — одна нога здесь, другая – там.)
  10. Может ли собака получить инфаркт? (Ответ — может, если ей создать человеческие условия.)

Загадки для взрослых — сложные


Сложные загадки для взрослых

Если в вашем доме бывают люди, которым по душе более сложные ребусы, тогда попробуйте их удивить и предложите разгадывать сложные загадки.

Загадки для взрослых — сложные:

  1. Один оборот вокруг Земли спутник делает за 1 час 40 минут, а другой — за 100 минут. Как это может быть? (Ответ — 100 минут – это и есть 1 час 40 минут.)
  2. Что не имеет длины, глубины, ширины, высоты, а можно измерить? (Ответ — время, температура)
  3. Если в 12 часов ночи идет дождь, то можно ли ожидать, что через 72 часа будет солнечная погода? (Ответ — нет, так как через 72 часа будет ночь.)
  4. Что может путешествовать по миру, оставаясь в одном и том же углу? (Ответ — почтовая марка на конверте.)
  5. Какой возраст у ребенка самый хороший? (Ответ — когда вы уже не водите его за руку, а он еще не водит вас за нос.)
  6. Что это, две руки, два крыла, два хвоста, три головы, три туловища и восемь ног? (Ответ — всадник, держащий в руках курицу.)
  7. На дереве сидело 10 птиц. Пришел охотник и подстрелил одну птицу. Сколько птиц осталось на дереве? (Ответ — ни одной, остальные испугались выстрела и улетели.)
  8. Товар сначала подорожал на 10%, а потом подешевел на 10%. Какова его стоимость сейчас относительно первоначальной? (Ответ — 99%: после подорожания к 100% прибавили 10% — получилось 110%; 10% от 110% = 11%; затем от 110% отнимаем 11% и получаем 99%.)
  9. Вы проехали на машине две трети пути. В начале пути бензобак машины был полон, а сейчас он заполнен на одну четверть. Хватит ли бензина до конца пути, при аналогичном расходе? (Ответ — нет, так как 1/4 <1/3.)
  10. Один глухонемой человек зашел в магазин канцтоваров, чтобы купить точилку для карандашей. Он воткнул себе палец в левое ухо и кулаком другой руки сделал вращательное движение около своего правого уха. Продавец сразу понял, о чем его просят. Потом в тот же магазин вошел слепой человек. Как он объяснил продавцу, что хочет купить ножницы? (Ответ — просто сказал, он ведь слепой, а не немой.)
  11. В подъезд одновременно зашли 2 человека. У одного квартира на 3-м этаже, у другого — на 9-м. Во сколько раз первый человек доедет быстрее второго? Примечание: они одновременно нажали кнопки в 2 лифтах, движущихся с одинаковой скоростью. (Ответ — обычный ответ: в 3 раза. Правильный ответ: в 4 раза. Лифты обычно ездят с 1-го этажа. Первый проедет 3-1=2 этажа, а второй 9-1=8 этажей, т.е. в 4 раза больше.)
  12. Один студент рассказывает другому: «Вчера баскетбольная команда нашего колледжа выиграла встречу по баскетболу со счетом 76:40. При этом ни один баскетболист в этом матче не забросил ни одного мяча. (Ответ — играли женские команды.)
  13. Мужчина заходит в магазин, покупает колбасу и просит ее нарезать, но не поперек, а вдоль. Продавщица спрашивает: «Вы пожарник?» — «Да». Как она догадалась? (Ответ — мужчина был в форме.)
  14. На одной одесской улице было три портняжные мастерские. Первый портной рекламировал себя так: «Лучшая мастерская в Одессе!» Второй — «Лучшая мастерская в мире!» Третий «переплюнул» их обоих. (Ответ — «Лучшая мастерская на этой улице!»)
  15. Если пять кошек ловят пять мышей за пять минут, то сколько времени нужно одной кошке, чтобы поймать одну мышку? (Ответ — пять минут.)

Загадки – обманки с подвохом про всё на свете

Утром рано по утру,
Всех разбудит … ©
(Неверный ответ – Кенгуру; Верный ответ – Петух, будильник)

***

Утром плотный завтрак нужен,
А вот днем полезен … ©
(Неверный ответ – Ужин; Верный ответ – Обед)

***

И капризна, и упряма,
В детский сад не хочет …
(Неверный ответ – Мама; Верный ответ – Дочка)

***

Взяв за ручку по утрам
Ведут до школы папы …
(Неверный ответ – Мам; Верный ответ – Детей)

***

Утром в каждом доме драма –
Кашу есть не хочет …
(Неверный ответ – Мама; Верный ответ – Ребенок)

***

На базар идут старушки
Покупать себе …
(Неверный ответ – Игрушки; Верный ответ – Продукты)

***

На прививки и уколы
Мамы деток водят в …
(Неверный ответ – Школу; Верный ответ – Поликлиннику)

***

В ползунках всегда одет
Спит в саду с пустышкой …
(Неверный ответ – Дед; Верный ответ – Малыш)

***

День рожденья на носу —
Испекли мы…
(Неверный ответ – Колбасу; Верный ответ – Торт)

***

На обед сыночку Ване
Мама варит суп в…
(Неверный ответ – Стакане; Верный ответ – Кастрюле)

***

Попросила мама Юлю
Ей чайку налить в…
(Неверный ответ – Кастрюлю; Верный ответ – Чашку)

***

Вся хрипит, чихает Лада:
Съела много…
(Неверный ответ – Шоколада; Верный ответ – Мороженого)

***

Холоднющее местечко
В доме нашем это …
(Неверный ответ – Печка; Верный ответ – Холодильник)

***

Просит бабушка Аркашу
Из редиски скушать …
(Неверный ответ – Кашу; Верный ответ – Салат)

***

Говорит нам папа басом:
«Я люблю конфеты с…
(Неверный ответ – Мясом; Верный ответ – Шоколадом, кремом…)

***

Чинят крыши, мебель, рамы,
На рыбалку ходят …
(Неверный ответ – Мамы; Верный ответ – Папы)

***

Я рисую акварелью,
Папа пилит доски …
(Неверный ответ – Дрелью; Верный ответ – Пилой)

***

Ни к чему весь этот спор —
Резать ткань бери…
(Неверный ответ – Топор; Верный ответ – Ножницы)

***

Куклам платьица, штанишки
Любят шить всегда…
(Неверный ответ – Мальчишки; Верный ответ – Девчонки)

***

Лет, наверно, двести
Петиной…
(Неверный ответ – Невесте; Верный ответ – Черепахе)

***

Красным лаком захотела
Я себе покрасить…
(Неверный ответ – Тело; Верный ответ – Ногти)

***

Гладить майку и трусы,
Берем горячие …
(Неверный ответ – Часы; Верный ответ – Утюг)

***

Всем домам нужна охрана,
Дверь не может быть без…
(Неверный ответ – Крана; Верный ответ – Замка)

***

И в Воронеже, и в Туле,
Дети ночью спят на…
(Неверный ответ – Стуле; Верный ответ – Кровати)

***

Белым снегом всё одето –
Значит, наступает …
(Неверный ответ – Лето; Верный ответ – Зима)

***

Все зимою любят дети,
Ездить на … ©
(Неверный ответ – Велосипеде; Верный ответ – Санках)

Загадки – обманки про профессии

К первоклашкам входит в класс
Лишь бесстрашный…
(Неверный ответ – Водолаз; Верный ответ – Учитель)

***

Черный весь, как-будто грач,
С нашей крыши лезет…
(Неверный ответ – Врач; Верный ответ – Трубочист)

***

Чинит в доме всё что можно
Очень мастерски … ©
(Неверный ответ – Художник; Верный ответ – Ремонтник)

***

Лекарства нам в аптеке
Продаст …
(Неверный ответ – Библиотекарь; Верный ответ – Фармацев)

***

На рояле, пианино
Вальс исполнит …
(Неверный ответ – Балерина; Верный ответ – Пианистка)

***

Ходят по клеточкам конь и ладья,
Ход свой победный готовит …
(Неверный ответ – Судья; Верный ответ – Шахматист)

***

Высадить новые саженцы в ельник
Снова отправится утром наш …
(Неверный ответ – Мельник; Верный ответ – Лесник)

***

Под куполом цирка в опасный полёт
Отправится смелый и сильный …
(Неверный ответ – Пилот; Верный ответ – Воздушный гимнаст)

***

Складки, карманы, оборки и кант –
Платье красивое сшил …
(Неверный ответ – Музыкант; Верный ответ – Портной)

***

Кто пасёт коров, овец?
Ну, конечно, …
(Неверный ответ – Продавец; Верный ответ – Пастух)

***

Булки нам и калачи
Каждый день пекут …
(Неверный ответ – Врачи; Верный ответ – Пекари)

***

Арий, опер сочинитель
Называется …
(Неверный ответ – Учитель; Верный ответ – Композитор)

***

В классе английский слышится говор –
Новую тему даёт детям …
(Неверный ответ – Повар; Верный ответ – Учитель)

Загадки – обманки про буквы, цифры и счет

Мы запомнили легко:
Номер первый – буква …
(Неверный ответ – О; Верный ответ – А)

***

Вы поверьте мне, друзья,
Что вторая буква … ©
(Неверный ответ – Я; Верный ответ – Б)

***

Буква третья, погляди,
Несомненно это … ©
(Неверный ответ – И; Верный ответ – В)

***

Круглой формы голова,
Той же формы буква … ©
(Неверный ответ – А; Верный ответ – О)

***

Завершает буквы все
В алфавите буква … ©
(Неверный ответ – Е; Верный ответ – Я)

***

Объяснял учитель Ире,
Что два больше, чем …
(Неверный ответ – Четыре; Верный ответ – Один)

***

Под деревом четыре льва,
Один ушёл, осталось …
(Неверный ответ – Два; Верный ответ – Три)

***

Ты на птичку посмотри –
Ног у птицы ровно …
(Неверный ответ – Три; Верный ответ – Две)

***

Вышел зайчик погулять,
Лап у зайца ровно …
(Неверный ответ – Пять; Верный ответ – Четыре)

***

Цветов не радуге не счесть,
А всего их ровно … ©
(Неверный ответ – Шесть; Верный ответ – Семь)

***

Нашёл пять ягодок в траве
И съел одну, осталось …
(Неверный ответ – Две; Верный ответ – Четыре)

***

Мышь считает дырки в сыре:
Три плюс две – всего …
(Неверный ответ – Четыре; Верный ответ – Пять)

***

На уроках будешь спать,
За ответ получишь …
(Неверный ответ – Пять; Верный ответ – Два)

Загадки – обманки про животных

Где немытая посуда
Там усач живет не худо,
Очень сложно истребим,
Всюду проползет … ©
(Неверный ответ – Пингвин; Верный ответ – Таракан)

***

Села птичка на дубу,
Завела свое «ку-ку»
Налетела вдруг гроза
Улетела … ©
(Неверный ответ – Стрекоза; Верный ответ – Кукушка)

***

Вдоль по листику ползет,
Бедный листик весь сгрызет.
У нее так много ног…
Кто же это ? … ©
(Неверный ответ – Осьминог; Верный ответ – Гусеница)

***

Жизнь их связана с трудом,
Их горой обычно дом,
Тащат в домик всё подряд
Дружный стройотряд …©
(Неверный ответ – Цыплят; Верный ответ – Муравьев)

***

Он пищит, когда летит!
А укус его зудит.
Пока осень не задула,
Всех кусает нас … ©
(Неверный ответ – Акула; Верный ответ – Комар)

***

В пруду, болоте иль реке
На одной она ноге,
Смотрит тщательно вокруг,
Лягушку хочет съесть … ©
(Неверный ответ – Паук; Верный ответ – Цапля)

***

Всё в нем статно и красиво,
Тело, ноги, хвост и грива,
Скачет словно ураган
По полям бежит … ©
(Неверный ответ – Баран; Верный ответ – Конь)

***

Чистый двор ей слишком скушен,
Ей бы поваляться в лужах,
Съест обед и хрюкнет сладко,
В лужу вновь бежит … ©
(Неверный ответ – Лошадка; Верный ответ – Свинья)

***

Ночью слышен громкий вой,
Он всегда голодный злой,
Чуть в капкан не угодил,
Злой, зубастый … ©
(Неверный ответ – Крокодил; Верный ответ – Волк)

***

Крылья пестрые расправит
И порхает и порхает,
Все любуемся мы ей,
Это крошка … ©
(Неверный ответ – Муравей; Верный ответ – Бабочка)

***

Двор и дом свой охраняет,
Чужаков не подпускает,
У него отличный нюх
В конуре живет … ©
(Неверный ответ – Петух; Верный ответ – Собака)

***

Мягко в джунглях он ступает
Никого не подпускает,
Его грива, как огонь,
Трон зверей возглавил… ©
(Неверный ответ – Конь; Верный ответ – Лев)

***

С пальмы вниз,
На пальму снова
Ловко прыгает…
(Неверный ответ – Корова; Верный ответ – Обезьяна)

***

В доме я ловлю мышей,
Развлекаю малышей.
Голос мой ласкает слух,
Называюсь я…
(Неверный ответ – Петух; Верный ответ – Кошка)

***

Сильный, смелый и игривый
Ходит зверь с косматьй гривой.
Прячьтесь , звери, поскорей,
Ходит грозный …
(Неверный ответ – Воробей; Верный ответ – Лев)

***

Посредине океана
Поднялась струя фонтана.
Без труда и без забот
Жизнь в воде проводит …
(Неверный ответ – Кот; Верный ответ – Кит)

***

Кукарекает спросонок
Этот славный …
(Неверный ответ – Поросенок; Верный ответ – Петух)

***

Зимой в берлоге
Видит сон
Лохматый, косолапый…
(Неверный ответ – Слон; Верный ответ – Медведь)

***

На заборе поутру
Кукарекал…
(Неверный ответ – Кенгуру; Верный ответ – Петух)

***

Над лесом солнца луч потух,
Крадётся царь зверей…
(Неверный ответ – Петух; Верный ответ – Лев)

***

Мама с сумкой скок да скок,
В сумке маленький сынок.
Вот так странная семья,
С сумкой прыгает …
(Неверный ответ – Свинья; Верный ответ – Кенгуру)

***

Кто напуган так лисой,
Длинноухий и косой..
Тут трусишке не до игр!
От лисы несётся…
(Неверный ответ – Тигр; Верный ответ – Заяц)

***

В теплой лужице своей
Громко квакал…
(Неверный ответ – Воробей; Верный ответ – Лягушка)

***

По сосне, как в барабан,
Застучал в лесу…
(Неверный ответ – Баран, кабан; Верный ответ – Дятел)

***

Пищу хоботом берёт
Толстокожий…
(Неверный ответ – Бегемот; Верный ответ – Слон)

***

Он не дружит с ярким светом,
Под землёй зимой и летом?
Носом грядки все изрыл.
Кто же это …
(Неверный ответ – Крокодил; Верный ответ – Крот)

***

Ква-ква-ква – какая песня!
Что быть может интересней,
Что быть может веселей?
Так поёт нам …
(Неверный ответ – Соловей; Верный ответ – Лягушка)

***

Клубком свернулся – ну-ка, тронь!
Со всех сторон колючий…
(Неверный ответ – Конь; Верный ответ – Ёж)

***

Чтоб лягушкой поживиться,
Стала в речку это птица.
На одной ноге весь день
Простоит в воде …
(Неверный ответ – Тюлень; Верный ответ – Цапля)

***

Вы меня узнайте сами –
По пескам хожу с горбами.
Ем побеги саксаула,
Потому что я …
(Неверный ответ – Акула; Верный ответ – Верблюд)

***

Умный, серый и свободный,
Я зимой всегда голодный.
А для зайцев я гроза,
Потому что я …
(Неверный ответ – Коза, лиса; Верный ответ – Волк)

***

Кто любит по ветвям носиться?
Конечно, рыжая…
(Неверный ответ – Лисица; Верный ответ – Белка)

***

Кто жильё нам охраняет?
Посторонний не пройдёт:
Испугается он лая.
На пороге верный …
(Неверный ответ – Кот; Верный ответ – Пёс)

***

Нам неизвестно до сих пор:
В чем есть его секрет,
Зверь, как волшебный светофор.
Свой изменяет цвет.
Зеленый, синий… Угадай,
Теперь он красный …
(Неверный ответ – Попугай; Верный ответ – Хамелеон)

***

У него большие уши,
Сам себя облил, как в душе,
Поднял хобот, затрубил
Толстоногий …
(Неверный ответ – Крокодил; Верный ответ – Слон)

***

Пятнистая, рыжая кошка
На ветках проводит весь день.
Под вечер идёт на охоту
Хитрый и быстрый …
(Неверный ответ – Олень; Верный ответ – Ягуар)

***

На лугу меня пасла
Внучка вместе с бабушкой.
Молочка я припасла,
Вот такая …
(Неверный ответ – Бабочка; Верный ответ – Корова)

***

Она под панцирем живет,
На жизнь свою не злится,
Когда куда-нибудь ползет,
То не спешит …
(Неверный ответ – Тигрица; Верный ответ – Черепаха)

***

Длиннее шеи не найдёшь.
Сорвёт любую ветку…
(Неверный ответ – Ёж; Верный ответ – Жираф)

***

Даешь по косточке в обед
Не отнимешь плошки:
Грозно лают и рычат
Озорные …
(Неверный ответ – Кошки; Верный ответ – Щенки, Собаки)

***

У него постель под елью,
Занесённая метелью.
Спит в берлоге без забот
Косолапый …
(Неверный ответ – Бегемот; Верный ответ – Медведь)

***

Хвостик тоненький крючком.
Роет землю пятачком.
В лужу лезет снова
Толстая …
(Неверный ответ – Корова; Верный ответ – Свинья)

***

Быстрее всех от страха
Несется ….
(Неверный ответ – Черепаха; Верный ответ – Заяц, мышь…)

***

Зверь на носу
Свой носит рог –
Ему не подставляйте бок:
Когда сердит, когда он зол,
Все по пути снесет …
(Неверный ответ – Козел, осел; Верный ответ – Носорог)

***

Любит он овсом питаться,
Дом его зовётся – хлев.
«Иго-го! Хочу на волю!
Отпустите!», – просит …
(Неверный ответ – Лев; Верный ответ – Лошадь, конь)

***

Я нектарчик припасу,
Деткам в соты унесу.
А жужжу я там и тут,
Потому что я …
(Неверный ответ – Верблюд; Верный ответ – Пчела)

Загадки – обманки про сказочных героев

Служить злодею были рады,
Кричат «банана» до упада,
Позитивные ребята,
Это желтые … ©
(Неверный ответ – Цыплята; Верный ответ – Миньоны)

***

Есть спасателей команда,
Всё у них отлично, складно,
Всем помогут, всех спасут,
Даже косточку найдут.
Ну ка мне скажите вслух,
Кто же это … ©
(Неверный ответ – Винни Пух; Верный ответ – Щенячий патруль)

***

Родом кажется из леса,
Днем красавица принцесса
Ночью цвет ее зеленый,
Огр Шрек в нее влюбленный,
И она его подружка
Вся зеленая … ©
(Неверный ответ – Лягушка; Верный ответ – Фиона)

***

Волшебство ее из рук
Заморозит все вокруг,
Сестру свою так любит нежно,
Её имя … ©
(Неверный ответ – Белоснежка; Верный ответ – Эльза Холодное сердце)

***

Она живет среди морей,
Она верна мечте своей,
Прекрасны волосы, улыбка,
Чуть в сети не попалась … ©
(Неверный ответ – Рыбка; Верный ответ – Русалочка Ариэль)

***

Этот сказочный герой
Он свирепый и большой,
Как дыхнет, так станет жарко,
Вон с хвостом летит … ©
(Неверный ответ – Русалка; Верный ответ – Дракон)

***

Черный плащ летучей мыши
Может он летать по крышам,
Он герой среди людей,
Почти бессмертный он … ©
(Неверный ответ – Кащей; Верный ответ – Бэтмен)

***

Паутины, как снаряд,
Красно – синий весь наряд.
Лицо всегда скрывает он,
Очень славный, добрый … ©
(Неверный ответ – Гном, слон; Верный ответ – Человек Паук)

***

Самый яркий из пиратов
Капитан и командир.
По морям плывет корабль,
Любит воду … ©
(Неверный ответ – Мойдодыр; Верный ответ – Капитан Джек Воробей)

***

Коли вдруг сломалось что-то
Это их теперь забота,
С инструментом рюкзачок
Ростом мелкий … ©
(Неверный ответ – Пятачок, паучок; Верный ответ – Фиксик)

***

К нам с луны свалился вдруг,
Необычный новый друг,
Кто же это угадай,
Может это … ©
(Неверный ответ – Попугай; Верный ответ – Лунтик)

***

В Простоквашино он жил
И с Матроскиным дружил.
Простоват он был немножко,
Звали песика … ?
(Неверный ответ – Тотошка; Верный ответ – Шарик)

***

Он один из всех котов,
С мышами в мире жить готов,
Но дружить с ними непросто,
Как зовут кота … ? ©
(Неверный ответ – Матроскин; Верный ответ – Леопольд)

***

Он сыграет на гармошке
У прохожих на дорожке.
Не возьму я как-то в толк,
Заяц это или … ?
(Неверный ответ – Волк; Верный ответ – Крокодил Гена)

***

Все узнает, подглядит,
Всем мешает и вредит.
Ей лишь крыска дорога,
Старушка, хитрая … ?
(Неверный ответ – Яга, Лиса; Верный ответ – Шапокляк)

***

Друг зверей и друг детей
Добрый доктор … ?
(Неверный ответ – Бармалей; Верный ответ – Айболит)

***

Потерял он как-то хвостик,
Но его вернули гости.
Он ворчлив, как старичок
Этот грустный … ?
(Неверный ответ – Пятачок; Верный ответ – Иа)

***

Он большой шалун и комик,
У него на крыше домик.
Хвастунишка и зазнайка,
А зовут его … ?
(Неверный ответ – Незнайка; Верный ответ – Карлсон)

***

Она песенки поёт
И орешки все грызет,
Смотрит вся толпа, дивится,
Это рыжая … ©
(Неверный ответ – Лисица; Верный ответ – Белка)

***

Три могучих молодца,
Днем и ночью без конца,
Защищают жизнь людскую,
Землю русскую родную.
Дружно вместе, как три брата
Вышли в поле … ©
(Неверный ответ – Поросята; Верный ответ – Три богатыря)

***

Много дней он был в пути,
Чтоб жену свою найти,
А помог ему клубок,
Его звали …
(Неверный ответ – Колобок, осьминог; Верный ответ – Иван-Царевич)

***

Бедных кукол бьет и мучит,
Ищет он волшебный ключик.
У него ужасный вид,
С бородою … ?
(Неверный ответ – Айболит; Верный ответ – Карабас-Барабас)

***

Он пиявок добывал,
Карабасу продавал,
Весь пропах болотной тиной,
Его звали … ?
(Неверный ответ – Буратино; Верный ответ – Дуремар)

***

Это что за очень странный
Человечек деревянный?
На земле и под водой
Ищет ключик золотой.
Сунул нос свой в котелок.
Кто же это … ?
(Неверный ответ – Колобок; Верный ответ – Буратино)

***

С голубыми волосами
И огромными глазами,
Эта куколка – актриса,
А зовут ее … ?
(Неверный ответ – Алиса; Верный ответ – Мальвина)

***

В Простоквашино жил он,
Пёс веселый …
(Неверный ответ – Артемон; Верный ответ – Шарик)

***

Знает взрослый и малыш,
Что Матроскин – это …
(Неверный ответ – Мышь; Верный ответ – Кот)

***

Сёстры девочку-бедняжку
Называли замарашкой.
Всё помоет, сварит кашу,
Вся в золе обычно …
(Неверный ответ – Маша; Верный ответ – Золушка)

***

Он гулял по лесу смело,
Но лиса героя съела.
На прощанье спел бедняжка.
Его звали … ?
(Неверный ответ – Чебурашка; Верный ответ – Колобок)

***

Из муки его слепили,
Словно солнышко, приметный.
По тропинке покатился
Кто, скажи … ?
(Неверный ответ – Кащей Бессмертный; Верный ответ – Колобок)

***

И красива, и мила,
Родилась в цветке она,
С ноготок фигурочка,
Зовут её … ?
(Неверный ответ – Снегурочка; Верный ответ – Дюймовочка)

***

Жил в бутылке сотни лет,
Наконец, увидел свет,
Бородою он оброс,
Этот добрый … ?
(Неверный ответ – Дед Мороз; Верный ответ – джин Старик Хоттабыч)

***

В дремучем лесу, на болоте
Вы её непременно найдёте.
Не рыбка она, не лягушка,
Она Водяного подружка.
Стройная фигурка,
Зовут ее … ?
(Неверный ответ – Снегурка; Верный ответ – Кикимора Болотная)

***

Внутри него водица,
С ним не хотят водиться,
А все его подружки –
Пиявки да лягушки!
Водорослями он зарос
Это дедушка … ?
(Неверный ответ – Мороз; Верный ответ – Водяной)

***

Есть еще в лесу один
Очень важный господин.
Охраняет он леса,
Все лесные чудеса,
Весь обросший, в палках, шишках,
Так его рисуют в книжках.
Знает он язык зверей,
А зовут его… ©
(Неверный ответ – Кощей; Верный ответ – Леший)

***

Он живет в глуши лесной,
Отрицательный герой.
Он костями громыхает
И в округе всех пугает.
Это что за старичок?
Очень гадкий …
(Неверный ответ – Пятачок, паучок; Верный ответ – Кощей Бессмертный)

***

Волка оседлал он,
И во весь опор,
Мчался от погони
С царевной …
(Неверный ответ – Черномор; Верный ответ – Иван Царевич)

***

Рассказал про Буратино,
Про Мальвину, про Пьеро,
Про Тортилу, Карабаса
Русский сказочник …
(Неверный ответ – Ш. Перро; Верный ответ – Алексей Толстой)

Загадки это те же стихи, при этом они еще более увлекательны за счет того, что ребенку в финале предстоит догадаться о чем или о ком идет речь. Все дети обожают это, именно поэтому загадки так активно используют на различных праздниках, утренниках, на новогодних и на других детских мероприятиях.

Источники

  • https://heaclub.ru/zagadki-dlya-vzroslyh-luchshaya-podborka-s-otvetami
  • http://zagadki-otvetami.ru/smeshnye-zagadki/
  • https://ot2do7.ru/413-zanimatelnye-zagadki-shutki-s-otvetami.html
  • https://super-positive.ru/zagadki-s-podvohom/
  • https://www.prazdnik-sam.ru/zagadki/obmanki-dlya-detei-s-podvohom.html

Загадки с подвохом с ответами

Кирпич весит 1 килограмм и еще столько сколько полкирпича.
Сколько весит кирпич?

(Решение: Мы не знаем сколько весит кирпич, пусть он будет х, тогда
х=1+х/2
х-х/2=1
0,5х=1
х=2)
Еще можно рассуждать так: если масса одного кирпича равна 1кг+полкирпича,
то масса двух кирпичей — 2кг и еще кирпич.
Отсюда ясно, что масса кирпича — 2кг.

Сто одежек и все без застежек.

(Сэконд хэнд)

Старый дед, во сто шуб одет, кто его раздевает, тот слезы проливает.

(Бомж)

Сам алый, сахарный, кафтан — зеленый, бархатный.

(Пьяный прапор)

Краса девица сидит в темнице, а коса — на улице.

((Премьер-министр Украины Юлия Тимошенко арес тована российскими властями в районе Тузлинской косы))

Без окон, без дверей — полна горница людей.

(Интернет-чат)

Зимой и летом — одним цветом.

(Черный пиар)

Висит груша, нельзя скушать.

(Тренажер Майка Тайсона)

Назовите слово, в котором 40 гласных.

(Сорока [сорок «а»])

Летел по небу бегемотик, а по земле за ним бежал охотник с ружьем.
Охотник выстрелил, и бегемотик упал на него. Кто жив остался?

(Слоник, потому что он вылетел позже)

Чем отличается публика в цирке от воздушного шарика?

((Воздушный шарик сначала надувают, потом пускают,
а публику сначала впускают, а потом надувают))

Чем человек отличается от паровоза?

(Паровоз сначала свистит, потом трогается,
а человек сначала тронется, а потом ходит и свистит)

Чем отличается молодой холостяк от старого?

(Молодой холостяк прибирается в своем доме,
чтобы пригласить женщину, а старый приглашает в свой дом женщину, чтобы она прибралась)

Что общего между деньгами и гробом?

(И то, и другое сначала заколачивают, а потом спускают)

Два гвоздя упали в воду. Как фамилия грузина?

(Заржавели)

Что это такое: синий, большой, с усами и полностью набит зайцами?

(Троллейбус)

Сколько горошин может войти в один стакан?

(Нисколько, так как горошины не ходят)

Что это такое: висит на стене и пахнет?

(Часы: в них кукушка сдохла)

Почему слоны не летают?

(По воздуху)


Чем лошадь отличается от иголки?

(На иголку сначала сядешь, потом подпрыгнешь,
а на лошадь сначала подпрыгнешь, потом сядешь)

Чем первый этаж отличается от девятого?

(С первого этажа упадешь: «Бух! — А-а!».
А с девятого «А-а! — Бух!»)

Чем кончаются день и ночь?

(Мягким знаком)

Чем отличается трактор от помидора?

(Помидор красный, а в тракторе дверь открывается на ружу)

Что это такое: летит, шуршит, а не шуршавчик?

(Брат шуршавчика)

Что это такое: в небе есть, в земле нет;
у бабы — две, у мужика — ни одной?

(Буква «Б»)

Домашнее животное, на «т» начинается.

(Таракан)

Домашнее животное, на «д» начинается.

(Два таракана)

Домашнее животное, на. «ы» начинается.

(Ышо один таракан)

Что это: сидит на окне, говорит по-французски?

(Француз)

Как правильно говорить: «Не вижу белый желток» или «Не вижу белого желтка»?

(Желток не может быть белым)

Какое слово начинается с трех букв «г»
и заканчи вается тремя буквами «я»?

(«Тригонометрия»)

Что это: стоит черное, на одной ноге?

(Одноногий негр)

Что это: стоит черное, на двух ногах?

(Два одноногих негра)

Что это: стоит черное, на трех ногах?

(Нет, это не три одноногих негра,
а всего лишь рояль)

Что это: черное — на четырех ногах?

(Одноногий негр за роялем)

Почему в Париже девки ходят рыжие?

(По земле)

Один глаз, один рог, но не носорог. Что это?

(Корова из-за угла выглядывает)

Что стоит посреди Земли?

(Буква «м»)

М оскву заводили, во что первый гвоздь колотили?

(В шляпку)

Как правильно сказать: у рыб нет зуб, у рыбов нет зубов или у рыбей нет зубей?

(У рыб есть зубы)

Кто это — маленький, в земле живет, на букву «щ» начинается?

(Щервячок)

Почему поп шляпу покупает?

(Потому что даром не дают)

Чем отличается вода от близнецов?

(Вода — Н2О, а близнецы — О, аж два!)

Маленький, серенький, на слона похож. Кто это?

(Слоник)

Сколько жирафов влезает в «Запорожец»?

(Три: хотя машина и четырехместная,
но одно место предназначено для водителя)

А сколько бегемотов влезает в «Запорожец»?

(Ни одного, потому что все места в «Запорожце» уже заняты жирафами)

Шли три туриста по лесу, и было у них в рюкзаке на троих три пирожка.
В первый день они съели один пирожок.
Во второй день они съели второй пирожок.
На третий день смотрят в рюкзак — пусто!
Куда же делся третий пирожок?

(Они его потеряли)

Двое знакомых ехали в Восточном экспрессе.
У одного была немецкая овчарка, а у другого — оте чественный чемодан.
Как-то раз они поссорились, и в пылу ссоры первый выкинул в окно чемодан,
а второй в отместку выбросил в окно собаку. После этого оба рванули стоп-кран.
Поезд остановился, они выбежали — и видят: к ним несется овчарка, а в зубах у нее… Что?

(Нет, не чемодан! Правильно: в зубах овчарка держит пирожок, который потеряли три туриста в лесу)

От чего утка плавает?

(От берега)


Шел охотник мимо башни с часами.
Достал ружье и выстрелил.
К уда он попал?

(В милицию)

Представьте себе, что из Москвы через центр Земли
насквозь прорыли шахту диаметром один метр и кинули в нее камень диаметром пять сантиметров.
Долетит ли он до противоположного конца шахты?

(Нет, потому что на глубине три метра его съест маленький зелененький камнеглот)

Сколько операций нужно сделать, чтобы поместить жирафа в обыкновенный холодильник?

(Нужно сделать три операции:
открыть дверцу холо дильника, засунуть туда жирафа, закрыть дверцу)

А сколько нужно сделать операций,
чтобы помес тить в холодильник бегемота?

(Нужно сделать четыре операции:
открыть дверцу, вытащить жирафа, засунуть бегемота и закрыть дверцу)

Лев созвал всех зверей на собрание.
Явились все, кроме одного. Что это за зверь?

(Это бегемот. Он же в холодильнике сидит)

Вам нужно пересечь широкую реку, которая кишит крокодилами. Как вы это сделаете?

(Вплавь. Ведь крокодилы-то все на собрании у льва)

Кто быстрее пробежит вокруг Кремля — жираф или бегемот?

(Жираф, потому что бегемот сидит в холодильнике и не может принимать участие в этом соревновании)

Мальчик с девочкой в траве что-то делали на «е».

(Ели землянику)

Что это: маленькое, зелененькое, на глубине три метра сидит и камни глотает?

(Это маленький зелененький камнеглот)

Что это: веселое, красненькое, с пятнышками, из грядки торчит?

(Еще проще: веселый краснопяточный грядкоторчатель)

Сколько требуется негров, чтобы похоронить человека?

(Пять. Четверо несут гроб, а пятый идет впереди с магнитофоном)

Около 40 миллионов человек занимаются ЭТИМ по ночам. Что ЭТО такое?

(Интернет)

С горки ползком, в горку бегом.

(Сопля)

Стоит баба на полу, приоткрыв свою дыру, Что это?

(Печка)

Встанет, до неба достанет. Что это?

(Радуга)

В зубах доска, в глазах тоска. Кто это?
(Мужик провалился в деревенский туалет)
Что нельзя съесть на завтрак?

(Обед и ужин)

Без рук, без ног на бабу скок!

(Коромысло)

Кто под проливным дождем не намочит волосы?

(Лысый)

Одно колесо и тысяча крыльев. Что это?

(Тачка с навозом и мухами)

Что такое: твердое в мягкое вставляется, и шарики рядом болтаются?

(Серьги)

Красный, длинный, 21?

(Трамвай)

Что такое синее золото?

(Любимая жена напилась)

Прыгает ловко и ест морковку? Кто это?

(Легкоатлет на диете)

Две женщины у забора:
одна приклеена, другая пришита…
Что с ними нужно сделать?

(Первую отодрать, вторую — отпороть)

Что возбуждается палочкой Коха?

(1) туберкулез:
2) жена Коха) )

Что объединяет горелый хлеб, утопленника и бере менную женщину?

(Не успели вытащить…)

Что нужно делать, когда видишь зеленого человечка?

(Переходить улицу)

Зеленое, в пятнах, прыгает.

(Десантник)

Что имеет голову, но не имеет мозгов?
(Сыр, лук, чеснок)Один глаз, один рог, но не носорог?

(Корова из-за угла выглядывает)

Какое слово начинается с трех букв «Г» и заканчивается тремя буквами «Я»?

(Тригонометрия)

Мимо башни с часами шёл охотник,
достал ружьё и выстрелил.
Куда он попал?

(В милицию)

В каком случае шесть детей, две собаки, четверо взрослых, забравшись под один зонтик, не намокнут?

(Если не будет дождя)

Эти три телезвезды хорошо известны каждому из нас.
Блондина зовут Степан, шатена зовут Филипп. А как зовут лысого?

(Хрюша)

Что состоит из одного слова,
Но имеет тысячи писем?

(Почта)

Что имеет две головы,
Четыре глаза, шесть ног и хвост?

(Ковбой верхом на коне)

Не лает, не кусается, а так же называется.

(@)

Встанет, до небу достанет.

(Радуга)

Что нельзя съесть на завтрак?

(Обед и ужин)

Кто родился,а еще не умер?

(Тот кот живет)

Кто всегда спит, не снимая туфлей?

(Лошадь)

Когда поп в портках крест выносит?

(Всегда)

Какое изобретение позволяет смотреть сквозь стены?

(Окно)

Мальчик упал с лестницы длиной 15 метров.
Но он не пострадал. Почему?

(Он упал с нижней ступеньки)

За что поп шляпу купил?

(За деньги)

Однажды, когда я завтракал,
Мое обручальное кольцо соскользнуло с пальца
И упало в полную чашку кофе,
Но кольцо не намокло.
Почему?

(В чашке был молотый кофе)

Отчего корова ложится?

(Не умеет сидеть)

Сидят три кошки,
Против каждой кошки —
Две кошки. Много ли всех?

(Три)

Три человека плыли в лодке.
Она перевернулась,
Но только у двоих намокли волосы.
Почему?

(Один из них был лысым)

Сколько букв в алфавите?

(Семь, посчитайте — А-Л-Ф-А-В-И-Т)


Как называется человек,
У которого не все пальцы на одной руке?

(Нормальный. У вас же пальцы на обеих руках)

Какое лучшее средство от перхоти?

(Облысение)

Первый человек является хозяином драгоценных камней,
Второй человек является хозяином любви,
Третий человек является хозяином лопаты,
Четвертый человек является хозяином большой палки.
Кто они?

(Короли в колоде карт)

Что это — большой, как слон,
Но вообще нисколько не весит?

(Тень слона)

Кто под проливным дождем не намочит волосы?

(Лысый)

В пустыне лежит мертвый мужчина. За плечами мешок, на поясе фляга с водой.
На многие километры вокруг нет ни единой живой души.
От чего умер человек и что в его мешке?

(Человек умер от удара об землю,
а в мешке — парашют, который не раскрылся)

В 12-этажном доме есть лифт.
На первом этаже живет всего 2 человека,
от этажа к этажу количество жильцов увеличивается вдвое.
Какая кнопка в лифте этого дома нажимается чаще других?

(Независимо от распределения жильцов по этажам, кнопка «1»)

Маленькое, желтенькое под кроватью лежит, на «З» начинается.

(10 рублевая монета. Почему на «З»? Закатилась…)

Что это такое: летит и блестит?

(Комар с золотым зубом)

Что у женщины на теле, у еврея на уме,
Применяется в хоккее и на шахматной доске?

(Комбинация)

На какой вопрос никто никогда не ответит «да» несоврав?

(Спящий, на вопрос: «Вы спите?»)

Когда человек бывает в комнате без головы?

(Когда высовывает ее из окна на улицу)

Когда козе исполнится семь лет, что будет дальше?

(Пойдет восьмой)

К берегу реки подходят двое мужчин.
У берега одна лодка, которая может выдержать только одного.
Оба мужчины переплыли на противоположный берег на лодке.
Как они это сделали?

(Они подошли к разным берегам)

Как много кубиков можно
Положить в пустую корзину?

(Один. После корзина будет не пуста)

По чему, когда захочешь спать, идешь на кровать?

(По полу)

Что такое: 90/60/90?

(Скорость при гаишнике)

Среднее арифметическое между велосипедом и мотоциклом?

(Мопед)

Что такое черное, когда она чистая,
И белая, когда она грязная?

(Школьная доска)

Висит на стене, зеленое и пищит.

(Селедка. Висит на стене потому,
что я ее туда повесил, зеленая потому, что я ее покрасил,
а пищит, чтобы никто не догадался)

Из какой посуды нельзя ничего поесть?

(Из пустой)

У отца Мэри пять дочерей. Четырёх из них зовут Чече, Чочо, Чуча, Чичя. Как зовут пятую дочь?

(Мэри)


У меня нет костей и нет ног,
Но если меня положить тепло,
Я скоро начну ходить. Кто я?

(Яйцо)

Какое слово всегда звучит неверно?

(Слово «неверно»)

Почему поп шляпу покупает?

(Потому что даром не дают)

Почему собака бегает?

(По земле)

Сколько горошин может войти в стакан?

(0, горошины не ходят)

В небе есть,на земле нет,
У бабы два,а у мужика не одной!

(Буква «Б»)

Сколько фруктов можно съесть натощак?

(Только один фрукт, следующие будут уже не натощак)

В лесу заяц. Пошел дождь. Вопрос: под каким деревом заяц спрячется?

(Под мокрым)

Ехал купец ел солёный огурец.
Одну половинку съел кому вторая досталась?

(Алёне)

Мужик каждый день ездит на лифте с 11 этажа на 1,
а когда возвращается с работы то едет с 1 на 7, а потом пешком. Почему?

(Потому что он карлик и не может дотянуться до 11 кнопки)

50 невероятно сложных загадок, которые заставят отгадывать самые острые дети

Если забавная загадка похожа на более сложную версию банальной шутки, то сложная загадка похожа на сложную головоломку, которая в конце награждает детей смехом. Помимо обучения наших детей сочувствию, сотрудничеству, терпению и тому подобному, мы также хотим задействовать их интеллект. Но это не значит, что за обеденным столом им нужно проверять словарный запас или полагаться исключительно на глупые шутки для смеха.Книги, задания и головоломки — все это забавные (акцент на fun ) способы бросить вызов детям и побудить их решать проблемы. Но вы не хотите оскорблять интеллект вашего ребенка, задавая ему головоломки, которые слишком легко решить. По мере того, как они становятся старше, вам придется усерднее работать, чтобы держать их в тонусе, разгадывая все более сложные загадки.

Обдумывая трудную загадку, которая, как вы уверены, поставит вашего ребенка в тупик, вы также должны подумать о том, как работает его мозг. Лучшая часть действительно сложных загадок заключается в том, что они награждают каждого ребенка, в зависимости от загадки, которую вы им даете: есть игры с числами для математиков, игра слов для любителей книг и абсурдные головоломки для творческих мыслителей.Ниже приведены лучшие сложные загадки с ответами, которые ваш ребенок может разгадать самостоятельно… с достаточным терпением.

  1. Загадка: Какие две вещи нельзя есть на завтрак?
    Ответ: Обед и ужин
  2. Загадка: Что такого хрупкого, что произнесение названия сломает его?
    Ответ: Тишина.
  3. Загадка: Кто может бегать, но никогда не ходит, имеет рот, но никогда не разговаривает, имеет голову, но никогда не плачет, имеет кровать, но никогда не спит?
    Ответ: Река
  4. Загадка: Что может заполнить комнату, но не занимает места?
    Ответ: Свет
  5. Загадка: Если ты уронишь меня, я обязательно сломаюсь, но улыбнись мне, и я всегда улыбаюсь в ответ.Что я?
    Ответ: Зеркало
  6. Загадка: Какая столица во Франции?
    Ответ : Буква F — единственная заглавная буква во Франции.
  7. Загадка: Чем больше возьмешь, тем больше оставишь. Кто они такие?
    Ответ: Шаги
  8. Загадка: Поворачиваю один раз, то, что вышло, не попадает. Поворачиваю снова, то, что есть, не вылезет. Что я?
    Ответ : ключ
  9. Загадка: Мужчина зовет свою собаку с противоположного берега реки.Собака переходит реку, не промокнув, не используя мост или лодку. Как?
    Ответ: Река замерзла.
  10. Загадка: Что ломается, но никогда не падает, а что падает, но никогда не ломается?
    Ответ: День и ночь
  11. Загадка: Что проходит через города и поля, но не движется?
    Ответ: Дорога
  12. Загадка: Я всегда голоден и умру, если не накормить, но все, к чему я прикоснусь, скоро станет красным.Что я?
    Ответ: Пожар
  13. Загадка: Тот, кто это делает, в этом не нуждается; тому, кто его покупает, это не нужно. Человек, который его использует, не может его ни видеть, ни чувствовать. Что это?
    Ответ: Гроб
  14. Загадка: Мужчина смотрит на картину в музее и говорит: «Братья и сестры, у меня их нет, но отец этого человека — сын моего отца». Кто на картине?
    Ответ: Сын мужчины
  15. Загадка: С острыми клыками сижу и жду; с пронзительной силой хрущу судьбу; хватание жертв, провозглашение мощи; физическое соединение одним укусом.Что я?
    Ответ: Степлер
  16. Загадка: У меня есть озера без воды, горы без камня и города без построек. Что я?
    Ответ: Карта
  17. Загадка: Что человек любит больше жизни, ненавидит больше смерти или смертельной розни; то, чего желают довольные люди; бедные имеют, богатые нуждаются; скупец тратит, расточитель копит, а всех несут в могилу?
    Ответ: Ничего
  18. Загадка : Мистери у г-жи Мастард шесть дочерей, и у каждой дочери есть брат. Сколько человек в семье Горчичных?
    Ответ : В семье девять горчиц. Поскольку у каждой дочери один и тот же брат, у нас шесть девочек, один мальчик, а также мистер и мисс Мастард.
  19. Загадка : Меня любят или ненавидят люди. Я меняю внешность и мысли людей. Если человек позаботится о себе, я поднимусь еще выше. Некоторые люди могут захотеть попытаться спрятать меня, но я покажу.Как бы люди ни старались, я никогда не проиграю. Что я?
    Ответ : Возраст
  20. Riddle : Только одного цвета, но не одного размера,
    Застрял внизу, но легко летает.
    Присутствует на солнце, но не под дождем.
    Не причиняет вреда и не чувствует боли.
    Что это?
    Ответ : Тень
  21. Загадка : Кто это с шеей и без головы, с двумя руками и без рук?
    Ответ : Рубашка
  22. Загадка : Если одиннадцать плюс два равняется одному, чему равны девять плюс пять?
    Ответ :
    11 часов плюс 2 часа = 1 час
    9 часов плюс 5 часов = 2 часа
  23. Загадка : Можете ли вы записать восемь восьмерок, чтобы они в сумме давали тысячу?
    Ответ : 888 + 88 + 8 + 8 + 8 = 1000
  24. Загадка : Его нельзя увидеть, нельзя почувствовать, нельзя услышать и нельзя понюхать.Он лежит за звездами и под холмами, И пустые дыры он заполняет. Оно приходит первым и следует после, Конец жизни и убивает смех. Что это?
    Ответ : Темнота
  25. Загадка: Какое английское слово сохраняет то же произношение даже после того, как вы уберете четыре из пяти букв?
    Ответ: Очередь
  26. Загадка: Три игральные карты подряд. Можете ли вы назвать их этими подсказками? Справа от короля стоит двое. Слева от лопаты находится бриллиант.Туз слева от сердца. Сердце слева от лопаты. Теперь определите все три карты.
    Ответ: Три игральные карты подряд. Можете ли вы назвать их этими подсказками? Справа от короля стоит двое. Слева от лопаты находится бриллиант. Туз слева от сердца. Сердце слева от лопаты. Теперь определите все три карты.
    Ответ: Бубновый туз, Червовый Король, Пиковая двойка.
  27. Загадка: Что такое, если дать одну, у вас будет либо две, либо ни одной?
    Ответ: Выбор.
  28. Загадка: Джордж, Хелен и Стив пьют кофе. Берт, Карен и Дэйв пьют газировку. Используя логику, Элизабет пьет кофе или газировку?
    Ответ: Элизабет пьет кофе. Буква E встречается дважды в ее имени, как и в именах других, пьющих кофе.
  29. Загадка: У меня есть ветви, но нет листьев, ствола и фруктов. Что я?
    Ответ: Банк.
  30. Загадка: У чего голова, хвост, коричневый, а ног нет?
    Ответ: Пенни.
  31. Загадка: Когда меня используют, я бесполезен. Однажды предложили, вскоре отвергли. Я часто не выражаю отчаяния. Что я?
    Ответ: Отговорка.
  32. Загадка: У меня четыре ноги, но нет волос. Люди катаются на мне часами, но никуда не едут. Мне всегда удается быть готовым к работе, без необходимости дергать или включать. Что я?
    Ответ: Стол
  33. Загадка: Какое из следующих слов не входит в группу и почему? CORSET, COSTER, SECTOR, ESCORT, COURTS
    Ответ: Суды.Все остальные являются анаграммами друг друга.
  34. Загадка: Когда Джону было шесть лет, он забил гвоздь в свое любимое дерево, чтобы отметить свой рост. Десять лет спустя, в возрасте шестнадцати лет, Джон вернулся, чтобы посмотреть, насколько выше был гвоздь. Если бы дерево росло на пять сантиметров каждый год, насколько выше был бы гвоздь?
    Ответ: Гвоздь будет на одной высоте, поскольку деревья растут на верхушках.
  35. Загадка: Я в четыре раза старше своей дочери.Через 20 лет я буду вдвое старше нее. Сколько нам сейчас лет?
    Ответ: Мне 40 лет, дочери 10 лет.
  36. Загадка: Был человек, который родился раньше своего отца, убил его мать и женился на его сестре. Тем не менее, в том, что он сделал, не было ничего плохого. Почему?
    Ответ: Его отец был впереди него, когда он родился, поэтому он родился раньше него. Его мать умерла при родах. Наконец, он вырос и стал священником и женился на своей сестре на ее церемонии.
  37. Загадка: Лилия — кувшинок в небольшом пруду. Лили удваивает свой размер каждый день. На 20-й день она покрывает весь пруд. В какой день Лили была размером с половину пруда?
    Ответ: День 19, это не 10, потому что на 20-й день она увеличилась вдвое по сравнению с 19-м днем, поэтому 19 должен быть половиной размера пруда.
  38. Загадка: Какой распространенный английский глагол становится своим прошедшим временем, переставляя свои буквы?
    Ответ: Ешь и ешь.
  39. Загадка: На вопрос, сколько ей лет, Сьюзи ответила: «Через два года я буду вдвое старше, чем была пять лет назад.» Сколько ей лет?
    Ответ: Ей 12.
  40. Загадка: Легче, чем то, из чего я сделан, Больше меня скрыто, чем видно. Что я?
    Ответ: Айсберг.
  41. Загадка: Он короче остальных, но когда ты счастлив, ты поднимаешь его, как лучший. Что это?
    Ответ: Большой палец.
  42. Загадка: Человек, купивший его, не хочет его себе. Человек, который его покупает, покупает это не для себя.И человек, которому это нужно, не знает, что это нужно ему. Что это?
    Ответ: Гроб.
  43. Загадка: У меня есть города, но нет домов. У меня есть горы, но нет деревьев. У меня есть вода, но нет рыбы. Что я?
    Ответ: Карта.
  44. Загадка: У девушки столько братьев, сколько сестер, но у каждого брата братьев вдвое меньше, чем сестер. Сколько братьев и сестер в семье?
    Ответ: Четыре сестры и три брата.
  45. Загадка: Сначала подумайте о цвете облаков. Затем подумайте о цвете снега. А теперь подумайте о цвете яркой полной луны. А теперь ответь быстро: что пьют коровы?
    Ответ: Вода
  46. Загадка: Разные огни делают меня странным, поэтому я буду менять их на разные размеры. Что я?
    Ответ : Я зрачок глаза.
  47. Загадка: У чего есть хвост и голова, но нет тела?
    Ответ : Монета
  48. Загадка: Я нечетное число.Забери письмо и я стану ровным. Какой я номер?
    Ответ: Семерка
  49. Загадка: Если красный дом сделан из красного кирпича, а желтый — из желтого кирпича, из чего сделана теплица?
    Ответ: Все теплицы стеклянные.
  50. Загадка: Что нельзя класть в кастрюлю?
    Ответ: Крышка.

Ой! Пожалуйста, попробуйте еще раз.

Спасибо за подписку!

66 действительно сложных загадок [для взрослых и детей]

Мы начинаем разгадывать загадки, когда мы очень молоды, но никогда не устаем от попыток отгадать ответ на загадку.По мере того, как мы становимся старше, мы можем разгадывать все более сложные загадки. Надеюсь, вы читаете эту статью, потому что готовы принять вызов. На всякий случай начнем с подготовки.

Советы по разгадыванию сложных загадок

  1. Хорошо подумайте, прежде чем пытаться ответить.
    Загадки составлены таким образом, что первый ответ, который вы думаете, обычно бывает неправильным. Цель состоит в том, чтобы ввести вас в заблуждение, представив очевидный ответ.Остановитесь и подумайте еще, прежде чем делать предположения.
  2. Практикуйтесь, разгадывая все более сложные загадки.
    Эта статья начинается с нескольких сложных, коротких, однострочных загадок. Затем у нас есть жесткое «Что я?» загадки. Далее мы предложим вам несколько длинных загадок. Наконец, у нас есть несколько сложных логических загадок, которые по-настоящему бросят вызов вашему разуму. Каждый раз, когда вы успешно отвечаете на загадку, ваша способность разгадывать загадки становится лучше.
  3. Ищите закономерности, подсказки и спрашивайте себя: «В чем на самом деле вопрос?»
    Вы заметите, что некоторые загадки основаны на буквах, а некоторые — на числах.Иногда ответ встроен прямо в загадку. Некоторые загадки основаны на двойном значении, например, таких слов, как улов и холл. Как только вы начнете замечать закономерности, разгадывать загадки станет проще.
  4. Разбейте загадку на части и ответьте на каждую из них.
    Более длинные загадки легче разгадывать, если разбить их на части и решить каждую. Это особенно хорошо подходит для логических загадок и загадок. Помните, что каждое слово и знак препинания важны.
  5. Повторение загадки может помочь вам найти решение.
    Прочитайте загадку несколько раз или попросите человека, который загадывает загадку, повторить ее несколько раз. Знакомство со словами в загадке помогает, и чем больше вы читаете загадку, тем больше вы склонны видеть подсказки, дающие ответ.

А теперь пора бросить вызов самому себе, разгадывая сложные загадки с ответами!

Короткие сложные загадки

Наши сложные, короткие, однострочные загадки отлично подходят для того, чтобы делиться ими с друзьями и семьей.Их легко запомнить, они забавны и помогают снять стресс. Эти короткие сложные загадки хорошо подходят для командной игры; посмотрим, какая команда может ответить на большее количество загадок. Или установите таймер, чтобы увидеть, какая команда самая быстрая.

  • Как превратить шесть в нечетное число?

    Удалите букву S, и вы получите IX, то есть 9 римскими цифрами.

  • Какую воду можно есть и жевать?

    Арбуз

  • У каких столов нет ножек?

    Периодические таблицы и таблицы умножения

  • Что вы получите, если сложите две ежевики и пять яблок?

    Дом, полный электронных устройств

  • Что такое пушистое, милое, что можно обнимать, что поднимается и опускается?

    Щенок в лифте

  • Бочка с водой весит 60 фунтов.Что нужно положить в него, чтобы он весил 40 фунтов?

    Ямка

  • Сколько секунд в январе?

    Только один — второе января

  • Что можно найти здесь, там и везде?

    Слово здесь

  • Что есть по одному в каждом углу и по два в каждой комнате?

    Буква О

  • В каком здании больше всего этажей?

    Библиотека

  • Что растет, а растет вниз?

    Гусь или утка

  • Что можно набить с пустыми руками?

    Перчатки

  • Какое слово выглядит одинаково в перевернутом и перевернутом виде?

    Плавание

  • Какое время суток пишется одинаково вперед и назад?

    Полдень

  • Моя голова красная, но становится черной, когда ее почесываешь.Что я?

    Совпадение

  • Каким словом описывается женщина, у которой не все пальцы на одной руке?

    Нормальный. Она бы не была такой, если бы все ее десять пальцев были на одной руке.

  • Что теряет голову утром, но возвращается ночью?

    Подушка

  • Что нельзя сжечь в огне или утопить в воде?

    Лед — он тает, а не горит в огне, и плавает в воде.

  • Что может заполнить всю комнату, не занимая места?

    Свет

  • Кто глухой, немой и слепой, но всегда говорит правду?

    Зеркало

  • Что на вкус лучше, чем запах?

    Язык

  • Как далеко может забежать собака в лес?

    Только на полпути, потому что после этого он бежит из леса.

  • Что можно сохранить после того, как кому-то дадут?

    Твое слово

  • Кто не может говорить, но всегда отвечает, когда к нему обращаются?

    Эхо

  • Что вы отвечаете, даже если оно никогда не задает вам вопросов?

    Дверной звонок или телефон

  • В каком слове из пяти букв осталась только одна после удаления двух букв?

    Stone

  • Законно ли мужчине в Калифорнии жениться на сестре своей вдовы?

    Нет, потому что жениться после смерти — незаконно.

  • Где океан самый глубокий?

    Внизу

«Что я?» Загадки — сложные

Вероятно, одними из самых первых загадок, которые вы выучили в детстве, были «Кто я?» загадки. Что ж, у нас есть для вас несколько хитрых. Увидев ответ, вы можете сказать себе: «Ну, это было легко». Однако это один из признаков хорошей загадки; сложно ответить, но тогда это кажется очевидным. Удачи с нашим трудом «Что я?» загадки.

  • Каждый день меня навещают многие люди со всего мира.Обычно они остаются всего на пару минут. Многие считают меня очень грязным, но мало кто захочет жить без меня. Когда люди приходят ко мне, они показывают часть себя, которую редко показывают другим. Что я?

    Туалет

  • Когда я понадоблюсь, вы выбросите меня.
    Когда я тебе не понадоблюсь, ты вернешь меня.
    Что я?

    Якорь

  • У меня много лиц, выражений и эмоций, и я всего в одном касании.
    Что я?

    Emojis

  • У меня есть филиалы.но у меня нет ни листьев, ни ствола, ни фруктов.
    Что я?

    Банк

  • Сначала вы выбрасываете внешний вид. Затем вы готовите внутри. И затем вы едите внешнее и выбрасываете внутреннее.
    Что я?

    Кукуруза

  • У чего есть голова, хвост, коричневый цвет и нет ног?

    Пенни

  • У меня четыре ноги, но нет волос.
    Люди катаются на мне часами,
    Но никуда не уходи.
    Мне не нужно тянуть или включать,
    я всегда могу быть готовым к работе.
    Что я?

    Стол

  • Слово, которое я знаю, шесть букв в нем,
    Уберите одну букву, а осталось двенадцать.
    Что я?

    Дюжины

  • Чем больше возьмешь, тем больше оставишь.
    Что я?

    Отпечатки пальцев

  • Если бы мужчина понес мою ношу,
    Он сломал бы себе спину.
    Я не большой, но оставляю серебро в своем роде.
    Что я?

    Улитка

  • От начала вечности до конца времени и пространства, до начала каждого конца и до конца каждого места.Что я?

    Буква E

  • Когда меня используют, я бесполезен,
    Однажды предложили, но вскоре отвергли.
    Я часто не в отчаянии.
    Что я?

    Отговорка

  • Я — слово из трех слогов, каждый из которых является словом.
    Моя первая статья — это широко используемая статья.
    Мой второй — млекопитающее необычайного интеллекта.
    Мой третий, хотя и не животное, используется для ношения.
    Собери меня воедино и я полезное искусство.
    Что я?

    Переносной корабль

  • Я закончил с лодками, с грузом, с грузами.
    В помещении я в некотором смысле узкая дорога.
    Что я?

    Перевозка / Холл

  • Меня обслуживают за столом,
    При сборищах из двух человек.
    Меня обслуживают маленькие, белые и круглые.
    Вы потеряете;
    Это часть удовольствия.
    Что я?

    Мячи для пинг-понга

  • Я в начале всего.
    Я в конце вечности.
    У меня центральный путь.
    Все мое воплощает безразличие.
    Что я?

    Апатия

  • Я могу трепетать, и у вас захватывает дух.
    Я выдержу избиение, но не синяк.
    Если я остановлюсь, вы обязательно проиграете.
    Я с тобой весь день и ночь.
    Что я?

    Your heart

Hard Logic Riddles

Наши сложные логические загадки, как следует из названия, представляют собой настоящий вызов. Обязательно остановитесь и хорошенько подумайте, прежде чем угадать ответ. Некоторые из них обязательно поставят вас в тупик.

  • Мужчина строит дом прямоугольной формы. Все стороны имеют южную экспозицию.Проходит большой медведь. Какого цвета медведь?

    Медведь белый, потому что дом построен на Северном полюсе.

  • У мистера Грина 4 сына. У каждого из его сыновей есть сестра. Сколько детей у мистера Грина?

    Четыре

  • Один большой фанат хоккея заявил, что может подсчитать счет перед любой игрой. Как он это сделал?

    Счет перед любой хоккейной игрой 0: 0.

  • Он короче остальных, но когда вы счастливы, вы поднимаете его, как лучший.Что это?

    Большой палец

  • Вы можете носить его с собой куда угодно, и он не станет тяжелым. Что это?

    Ваше имя

  • Водитель автобуса ходит не в ту сторону на улице с односторонним движением. Он проходит мимо копов, но они его не останавливают. Почему?

    Он шел

  • Что падает, но не ломается? Что ломается, но не падает?

    День и ночь

  • Это насекомое, и первая часть его имени является названием другого насекомого. Что это?

    Beetle

  • Есть лодка с прикрепленной к ней лестницей.Высота лестницы восемь футов. Если вода поднимется на четыре фута, какая часть лестницы будет на поверхности воды?

    Восемь футов, потому что к лодке прикреплена лестница.

  • Ходят по живым, они даже не бормочут. Ходят по мертвым, они бормочут и ворчат. Кто они такие?

    Leaves

  • Человек, который это придумал, не хочет этого для себя. Человек, который его покупает, покупает это не для себя. И человек, которому это нужно, не знает, что ему это нужно. Что это?

    Гроб

  • Человек проезжает мимо и видит три двери: алмазную дверь, рубиновую дверь и изумрудную дверь.Что он открывает первым?

    Дверь его машины

  • Джим и Том решили сыграть в теннис друг против друга. Они ставят 1 доллар на каждую сыгранную игру. Джим выиграл три ставки, а Том выиграл 5 долларов. Сколько игр они сыграли?

    Одиннадцать игр

  • Как сложить восемь восьмерок, чтобы получить число 1000?

    888 + 88 + 8 + 8 + 8 = 1000

Загадки сложной истории

Многие решатели загадок считают загадки сложной истории не только самыми сложными, но и самыми увлекательными.Помните, что если вы решите рассказать загадку, вам не нужно делать это дословно. Вы можете варьировать имена и историю, если есть подсказки в правильном порядке. Посмотрите, как можно справиться с нашими сложными загадками.

Самый умный принц

Король хочет, чтобы его дочь вышла замуж за самого умного из трех чрезвычайно умных молодых принцев, и поэтому мудрецы короля разработали тест на интеллект.

Князья собираются в комнате и сидят лицом друг к другу.Им показаны две черные шляпы и три белые шляпы. Им завязывают глаза и надевают на каждую голову по одной шляпе. Остальные шляпы спрятаны в другой комнате.

Король говорит трем принцам, что первый принц, который определит цвет его шляпы, не снимая ее и не глядя на нее, женится на его дочери. Неправильная догадка будет означать смерть.

Повязки сняты. Вы один из принцев. Вы видите две белые шляпы на головах другого принца. Через какое-то время вы понимаете, что другие принцы не понимают, какой цвет их шляпы, или боятся угадать.Поразмыслив, вы сообщаете королю цвет своей шляпы. Что это?

Вы полагаете, что ваша шляпа белая, и это правильно.

Король не выбрал две белые шляпы и одну черную шляпу. Это означало бы, что два принца увидят одну черную шляпу и одну белую шляпу. Вы были бы в невыгодном положении, если бы были единственным принцем в черной шляпе.

Если бы вы носили черную шляпу, одному из других принцев не потребовалось бы много времени, чтобы сделать вывод, что он носит белую шляпу.

Если бы разумный принц увидел белую шляпу и черную шляпу, он в конце концов понял бы, что король никогда не выберет две черные шляпы и одну белую шляпу.Любой принц, увидевший две черные шляпы, сразу узнал бы, что на нем белая шляпа.

Если принц видит одну черную шляпу, он может решить, что он одет в белую.

Следовательно, единственное честное испытание для всех трех князей — быть в белых шляпах.

История капитана

Капитан корабля рассказал интересную историю.

«Я путешествовал по океанам вдоль и поперек. Однажды двое моих моряков стояли по разные стороны корабля. Один смотрел на запад, а другой на восток.И в то же время они могли ясно видеть друг друга. Вы можете сказать мне, как это было возможно? »

Матросы стояли спиной к бортам корабля и смотрели друг на друга.

Горох и чечевица

Бедный фермер пошел на рынок, чтобы продать свой горох и чечевицу. Однако у него был только один мешок, и он не хотел смешивать горох и чечевицу. Итак, он сначала насыпал горох, завязал мешок посередине, а затем заполнил верхнюю часть мешка чечевицей.

На рынке появился трактирщик со своим пустым мешком. Он хотел купить горох, но не хотел чечевицу.

Засыпать семена в другом месте, кроме мешков, было невозможно. Торговые мешки не работали, да и крестьянин не хотел проделывать дыру в своем мешке.

Как фермер перенес горох в мешок трактирщика?

Вылейте чечевицу в мешок трактирщика, завяжите его и выверните наизнанку. Всыпать горошек. Затем развяжите мешок и вылейте чечевицу обратно в мешок фермера.

Побег из тюрьмы

Молодой человек арестован во время отпуска за незначительное нарушение. Его поместили в камеру с земляным полом и одним окном. Хотя в окне нет решеток, оно слишком высоко, чтобы он мог дотянуться до него. В камере есть только лопата. Он знает, что не получит ни еды, ни воды. У него всего два дня, чтобы сбежать, иначе он умрет. Человек не может рыть туннель, потому что на это у него уйдет больше двух дней. Как он выберется из камеры?

Он использует лопату, чтобы создать кучу грязи под окном, чтобы он мог забраться на нее и выбраться из камеры.

Школьное убийство

В первый день учебы кто-то убил учителя английского языка. Полиция подозревала, что это сделали четыре человека в школе: дворник, учитель истории, тренер по баскетболу и директор. Это были их алиби:

  • Дворник сказал, что убирал ванные комнаты.
  • Учитель истории сказал, что проводит промежуточный тест.
  • Тренер по баскетболу сказал, что он проводил тренировочные упражнения со своими игроками.
  • Директор сказал, что был в своем офисе.

Представив алиби, полиция немедленно арестовала убийцу. Кто убил учителя английского языка и откуда об этом узнала полиция?

Учитель истории убил учителя английского языка. Он сказал, что проводил промежуточный тест, но это был первый день в школе.

Ядовитые напитки

Мария и Джудит однажды вечером после работы вместе пошли выпить. Они заказали такой же напиток. Джудит очень хотела пить и пила больше, чем Мария.Она допила пять рюмок за время, которое потребовалось Марии, чтобы допить только одну.

Напитки были отравлены, но умерла только Мария. Почему?

Яд был во льду. Поскольку лед Марии успел растаять, она была отравлена, а Юдифь — нет, хотя на следующее утро у нее было ужасное похмелье!

Любовь и похороны

София, молодая девушка была на похоронах своей матери. Она встретила милого молодого человека, которого никогда раньше не видела, и после службы они провели немного времени вместе.Потом она занялась другими людьми и не узнала ни его имени, ни номера телефона, прежде чем он ушел. Она приложила все усилия, чтобы найти его, но никто не знал, кто он такой и как с ним связаться. Через несколько недель ее старшая сестра умирает, и полиция подозревает убийство. Кто убил сестру?

София убила свою сестру. Она надеялась, что если кто-то из ее семьи умрет, молодой человек, которого она встретила на похоронах матери, снова появится

Ну, вот и все; множество сложных загадок, чтобы бросить вызов даже самому опытному загадочнику.Поделитесь короткими сложными загадками с друзьями. Проверьте своих друзей и семью с помощью сложных логических загадок и вопроса «Что я?» загадки. И не забывайте загадки рассказов. Возможно, вам будет интересно превратить некоторые загадки, не связанные с рассказом, в загадки-рассказы. Просто добавьте имена, детали и подсказки и создайте воображаемый сценарий. Удачи и загадки!

Сьюзен изучала английский язык с двойным вторым по специальности «гуманитарные науки и бизнес» в Университете штата Аризона и получила степень магистра управления образованием в Университете Либерти.Она преподавала с четвертого по двенадцатый классы как в государственных, так и в частных школах. Предметы включали английский язык, американскую и всемирную историю и географию, математику, землю и физические науки, Библию, информационные технологии и творческое письмо.

Сьюзан писала-фрилансером более десяти лет, за это время она написала и отредактировала книги, газетные статьи, биографии, книжные обзоры, руководства, описания районов для риэлторов, презентации Power Point, резюме и множество других проектов.

Прочитать полную биографию

ДЕЙСТВИТЕЛЬНО СЛОЖНЫЕ ЗАГАДКИ с ответами для детей

Следующая подборка состоит из известных и оригинальных загадок с ответами, передаваемых из уст в уста в течение многих лет , их создатели анонимны.

Этот популярный контент более сложен, чем простых загадок и сложных загадок для решения , собранных в других разделах. Их разрешение представляет собой более высокий шаг для продолжения обучения. Вам нужно будет выяснить, какой общий объект описан в заявлении.

Готовы разгадать эти забавные, но очень сложные загадки с ответами? Не бойтесь. Их решение станет для вашего мозга увлекательной задачей. Вы можете решить их самостоятельно, или поиграть с этими чрезвычайно сложными загадками в группе , чтобы сделать игру еще более увлекательной. В соревновании вы будете стремиться придумать ответ раньше всех, что сделает его еще более увлекательным.

Поразмыслив над ними в течение нескольких минут, вы можете, , проверить ответы на эти очень сложные загадки , используя кнопку решения, включенную для каждой из них.Вы также найдете еще одну кнопку, которая даст вам подсказку, которая даст вам знать, на правильном ли вы пути. Мы советуем, прежде чем использовать любую из двух кнопок, внимательно прочитайте и постарайтесь отсеять элементы, которые там отвлекают, и поразмышлять над тем, что вы поняли при чтении. Легкий путь всегда менее полезен.

После того, как вы отгадали очень сложные загадки с ответами, вы можете запомнить их, чтобы поделиться ими со своими близкими или продемонстрировать свои знания в школе или в парке.

Кроме того, позже вы сможете продолжить развлекаться с другими разделами загадок, организованными по темам, или открыть для себя весь интересный контент, который мы предоставляем в ваше распоряжение, что вам также понравится.

Игра в эти чрезвычайно сложные загадки даст вам новый словарный запас, укрепит ваши навыки логического мышления и концентрации, а также пробудит вашу изобретательность. Вы хотите получить все эти преимущества?

Давай! Итак, нажмите кнопку «Подумайте» [это просто образно ;-), на случай, если вы не знали], потому что мы собираемся разгадывать несколько неразрешимых загадок, которые сложно разгадать .Посмотрим, кто их первым получит. Готовы к игре? Посмотрите, насколько вы умны в разгадывании очень сложных загадок.

100 самых сложных загадок 2021 года для детей и взрослых, чтобы проверить ваш гений

Pexels

Загадочник Джима Керри, возможно, носил культовый уродливый костюм в Batman Forever, , но он оставил нам запоминающуюся цитату «Загадывай мне эту загадку, загадывай мне то». Забавно, но лучше всего использовать эту фразу во время семейной игры, когда вы достаете наш список самых сложных загадок, которые когда-либо собирались.Загадайте своих друзей и семью загадками, которые варьируются от смешных до задумчивых и всего, что между ними.

В конце концов, головоломки хороши не только для того, чтобы рассмеяться, когда вы наблюдаете, как ваши близкие крутят колеса. Загадки тренируют обе стороны вашего мозга, что помогает ему дольше сохранять остроту. Поскольку они требуют нестандартного мышления, они также улучшают важные когнитивные навыки, такие как решение проблем и способность учиться.

Имея это в виду, прочтите этот список загадок и посмотрите, сколько у вас получится! Все, что выше 20, означает, что вы сертифицированный гений (или, по крайней мере, действительно, действительно умный) в наших книгах.Нет, правда. Вот как их сложно взломать. Ознакомьтесь с нашими любимыми сложными загадками 2020 года ниже.

1. Что начинается с Т, заканчивается на Т и в нем есть Т? Чайник.
2. Какое английское слово сохраняет то же произношение даже после того, как вы уберете четыре из пяти букв? Очередь.
3. Когда я понадоблюсь, вы меня выбросите. Когда я тебе не нужен, ты возвращаешь меня. Что я? Якорь.
4. Ты не хочешь меня, когда меня нет у тебя, но когда у тебя есть я, ты не хочешь потерять меня.Что я? Иск.
5. Какое слово в английском языке означает следующее: первые две буквы означают мужчину, первые три буквы означают женщину, первые четыре буквы означают великую, а весь мир означает великую женщину. Что за слово? Героиня.
6. Что получится, если добавить две ягоды ежевики и пять яблок? Дом, полный гаджетов.
7. Чем всегда все кончается? Буква «Г»
8. Я нечетное число. Забери письмо и я стану ровным. Какой я номер? Семь.
9. Ходите по живым, они даже не бормочут. Ходят по мертвым, они бормочут и ворчат. Кто они такие? Листья.
10. Чем больше у вас его, тем меньше вы видите. Что это? Тьма.
11. Можете ли вы назвать три дня подряд, не используя воскресенье, среду и пятницу? Вчера, сегодня и завтра.
12. Если красный дом сделан из красного кирпича, а желтый — из желтого кирпича, из чего сделана теплица? Стекло, все теплицы из стекла.
13. Что следующее в этой последовательности: JFMAMJJASON? Буква «Д.Последовательность содержит первую букву каждого месяца.
14. Какие из следующих слов не входят в группу и почему: CORSET, COSTER, SECTOR, ESCORT, COURTS? Суды. Все остальные являются анаграммами друг друга.
15. Мужчина и его начальник имеют одни и те же родители, но не являются братьями и сестрами. Как это возможно? Он работает не по найму.
16. Что имеет четыре пальца и большой палец, но не живое? Перчатка.
17. Что такое легкое, как перышко, но даже самый сильный человек в мире не может удержать его так долго? Дыхание.
18. Что имеет сердце, которое не бьется? Артишок.
19. Что нельзя использовать, если оно не сломано? Яйца.
20. Куда коровы отдыхают? Му-Йорк.
21. Кто я, когда сегодня позавчерашний день? Словарь.
22. Что имеет золотые волосы и что очищает после тебя? Метла.
23. Что движется не видя и плачет без глаз? Облако.
24. Что падает, но никогда не поднимается? Дождь.
25. Круглый, как яблоко, глубокий, как чаша, но все царские кони не могут его поднять.Что это? Хорошо.
26. На какой вопрос ты никогда не сможешь ответить утвердительно? Ты еще спишь?
27. Что делает вас молодым? Добавляем буквы «нг».
28. Я выхожу из шахты и меня всегда окружает лес. Меня все используют. Что я? Карандашный грифель.
29. Что может заполнить всю комнату, не занимая места? Свет.
30. У чего одна голова, четыре ноги и одна ступня? Кровать.
31. Что сделано из воды, но умрет, если вы положите его в воду? Кубик льда.
32. Ложусь только один раз — когда умру.Что я? Дерево.
33. У меня четыре ключа, но нет замков. У меня есть место, но нет места. Вы можете войти, но не можете войти внутрь. Что я? Клавиатура.
34. Что жвачка сказала туфле? Я застрял на тебе.
35. Что нельзя класть в кастрюлю? Его крышка.
36. Дай мне еды, и я буду жить. Дай мне воды, и я умру. Что я? Огонь.
37. Что дырявое, но может удерживать много воды? Губка.
38. Не моя сестра и не мой брат, но все же ребенок моих матери и отца.Кто я? Сам.
39. Что всегда перед вами, но чего не видно? Будущее.
40. Каждый раз, когда вы что-то теряете, вы всегда находите это в самом последнем месте, куда бы ни заглянули. Почему это? Это последнее место, куда вы бы посмотрели, потому что, как только вы его найдете, нет необходимости продолжать искать.
41. Ты купил меня на обед, но никогда не ешь меня. Что я? Столовые приборы.
42. Что можно измерить, но не увидеть? Время.
43. Сколько букв в алфавите? В «алфавите» одиннадцать букв.
44. Что может сделать слон такого, чего не может сделать никакое другое животное? Слоненок.
45. Я могу писать, я могу говорить, я могу быть разоблачен, я могу сломаться. Что я? Новости.
46. Понюхайте меня, купите и избавьте меня. Я не изменюсь. Что я? Аромат, цент и отправлено.
47. Как футболисты сохраняют хладнокровие во время игры? Стоят рядом с болельщиками.
48. Какое платье нельзя носить ни в коем случае? Адрес.
49. Что разбивается о воду, но никогда о суше? Волна.
50. Что остается? Будильник.
51. Мои кольца не из золота, но с возрастом я получаю их все больше. Что я? Дерево.
52. Что в конце радуги? Буква W.
53. Чем больше возьмешь, тем больше оставишь. Кто они такие? Шаги.
54. Что имеет десять букв и начинается с газа? Автомобиль.
55. Сколько секунд в году? 12 — 2 января, 2 февраля, 2 марта, 2 апреля…
56. Как камень в дереве, я помогу твоим словам пережить тебя. Но если вы толкаете меня, когда я стою, чем больше я двигаюсь, тем меньше я становлюсь.Что я? Карандаш.
57. Что будет черным, когда вы его получите, красным, когда вы его используете, и белым, когда вы его полностью освоите? Уголь.
58. Ты обезглавливаешь меня ножом, но плачешь надо мной после моей смерти. Что я? Лук.
59. Какая птица может поднять больше всего? Кран.
60. Я появляюсь дважды в вечности, и я всегда в пределах видимости. Что я?
61. Что можно держать правой рукой, но никогда — левой? Твоя левая рука.
62. Что можно набить с пустыми руками? Перчатки.
63. Что сильнее стали, но солнца боится? Лед.
64. Не успел произнести, чем сломался. Что это? Тишина.
65. Вперед я тяжелый, назад нет. Что я? Тон.
66. У кого 12 лиц и 42 глаза? Пара игральных костей.
67. Как заставить исчезнуть номер один? Добавьте букву G, и она исчезнет.
68. Сколько кирпичей нужно, чтобы построить кирпичное здание? Один кирпич — «достроить здание».
69. У чего есть шея, но нет головы? Бутылка.
70. Как можно сжечь яблоко, чернику, розу и тыкву, не оставив пепла, но сохранив его запах? Зажигание ароматических свечей.
71. Какое пятибуквенное слово становится короче, если к нему добавить две буквы? Короче. (Short + ‘er’)
72. Я могу жить только там, где есть свет. Но если на меня будет светить, я умру. Что я? Тень.
73. Что на вкус лучше, чем запах? Твой язык.
74. Что может пройти сквозь стекло, не разбив его? Свет.
75.Что летает, когда рождается, лжет, когда жив, и бежит, когда мертв? Снежинка.
76. Что можно бегать, но не ходить? Воды.
77. Что начинается, но не имеет конца и является концом всего, что начинается? Смерть.
78. Какое английское слово состоит из трех последовательных двойных букв? Бухгалтер.
79. Что можно увидеть в середине марта и апреле, чего нельзя увидеть в начале или в конце каждого месяца? Буква «Р».
80. В какое время начинается и заканчивается буква «N»? Полдень.
81. Какое слово начинается и заканчивается на Е, но состоит только из одной буквы? Конверт.
82. Где можно найти города, поселки, магазины и улицы, но не людей? Карта.
83. Что закапываешь живым и выкопаешь мертвое? Растение.
84. Что все время лежит на полу, но никогда не пачкается? Твоя тень.
85. Что такое хрупкое, что при произнесении названия его ломает? Тишина.
86. У кого есть руки, но не может хлопать? Часы.
87. У чего есть ветви и листья, но нет коры? Библиотека.
88. Какое дерево можно носить в руке? Пальма.
89. От чего никогда не избавиться, если на самом деле это потеряно? Ваш характер.
90. Что имеет один глаз, но не видит? Игла.
91. Что можно оставить после того, как кому-то подаришь? Твое слово.
92. Почему остров похож на букву Т? Потому что он посреди воды.
93. Откуда вы знаете, что морковь полезна для глаз? Вы никогда не увидите кроликов в очках.
94. Что вы можете сделать такого, чего не видите? Шум.
95. Что связывает двух людей, но трогает только одного? Обручальное кольцо.
96. Что падает, но не ломается? Что ломается, но не падает? День и ночь.
97. Кто женился на многих женщинах, но никогда не был женат? Священник.
98. У чего есть голова и хвост, но нет тела? Монета.
99. Если брат, его сестра и их собака не были под зонтиком, почему они не промокли? Дождя не было.
100. У чего есть зубы, но нельзя есть? Гребень.

30 невероятных загадок, которые очень сложно решить (с ответами) ▷ Туко.co.ke

Вы любите разгадывать головоломки? Человеческая природа — искать решение и понимание сложного вопроса, и невозможные загадки — идеальные головоломки. Может быть, вы хотите потренировать свой ум или просто скоротать время, ломая голову над неразрешимой головоломкой. Хотя интеллектуальные шутки — это весело, их легко решить за секунды. Однако разгадывание загадок занимает немного больше времени, что делает игру еще более увлекательной. При регулярной практике и последовательности загадки могут стать незаменимыми инструментами жизни.Сколько подготовки и оттачивания нужно, чтобы решить сложнейшую головоломку? Возможно, вам придется попытаться разгадать сложные загадки, чтобы разгадать это.

Изображение: pexels.com, (изменено автором)
Источник: UGC

«Разгадай мне это, дай мне загадку…». Такие слова являются синонимами загадок, в основном они используются как вступительные слова. И вам лучше подготовиться к творческому размышлению над загадкой, если вы надеетесь дать соответствующий ответ. Не забывайте, какими бы сложными они ни были, они предназначены для людей.

Читайте также

Лучшие цитаты гангстеров о любви, уважении и жизни

Почти невозможные загадки — рай для тех, кто ищет увлекательные испытания. Такие головоломки просты, но, поскольку они имеют скрытый смысл, многие люди попадают в тупик. Если вы верите в свои методы разгадывания загадок, поздравляю вас и добро пожаловать в путешествие по самым крутым загадкам.

Сборник невозможных загадок — проверьте свои навыки

Вы ищете невозможные загадки с простыми ответами? Конечно, вы весело проводили время, решая самые сложные из них, но хотите получить что-то более сложное и готовое решение, если оно окажется неразрешимым.

Они могут быть вам интересны, чтобы отточить свои навыки решения загадок и стать гуру. Независимо от вашего интереса, вы будете получать удовольствие от идеальных интеллектуальных задач, и неважно, молод вы или стары — эти загадки для тех, кто ищет головорезов.

1. Убийца приговорен к смертной казни. Ему предстоит выбрать одну из трех комнат: первая полна бушующих огней; второй — убийцы с заряженным ружьем; и третьи львы, которые годами не ели.Какая комната самая безопасная?

Читайте также

Популярные цитаты о ненавистниках, ненавидящих ваши отношения

Ответ:

Бедные львы умерли от голода.

2. На рыбалку ходили мальчик и врач. Мальчик был сыном доктора, но доктор не был отцом мальчика. Кто доктор?

Ответ:

Мать мальчика.

3. Что имеет четыре пальца и большой палец, но не живое?

Ответ:

Перчатка.

4. Если у человека пчела в руке, то что у него в глазу

Ответ:

Красота, потому что красота в глазах «пчеловода».

5. Две девушки вместе ужинали. Они оба заказали холодный чай. Одна девушка выпила их очень быстро и закончила за время, которое потребовалось другой, чтобы выпить только одну. Девушка, выпившая одну, умерла, а другая выжила. Все напитки были отравлены. Как выжила девушка, которая больше всего пила?

Ответ:

Яд был во льду, когда она медленно выпила одну рюмку, тающий лед высвободил яд.

Читайте также

Лучшие цитаты Джона Дикаря

ЧИТАЙТЕ ТАКЖЕ: Веселые каламбуры, цитаты и поговорки о пицце

6. Тот, кто меня заставляет, не говорит мне. Кто меня забирает, меня не знает. Кто меня знает, не хочу меня. Что я?

Ответ:

Фальшивые деньги.

7. Как вы называете комнату без окон и дверей?

Ответ:

Гриб.

8. Каждую ночь мне говорят, что делать, и каждое утро я делаю то, что мне говорят.Но все же я не ухожу от твоих ругательств.

Ответ:

Будильник.

9. Я то, что люди любят или ненавидят. Я меняю внешность и мысли людей. Если человек позаботится о себе, я поднимусь еще выше. Некоторых я обманываю. Для других я загадка. Некоторые люди могут захотеть попытаться спрятать меня, но я покажу. Как бы люди ни старались, я никогда не проиграю. Что я?

Ответ:

Возраст.

10.Его нельзя увидеть, нельзя почувствовать, нельзя услышать и нельзя понюхать. Он лежит за звездами, под холмами и пустыми ямами, которые он заполняет. Он приходит первым и следует за ним, заканчивает жизнь и убивает смех. Что это?

Читайте также

60+ цитат и высказываний о мести

Ответ:

Темнота.

11. Вы измеряете мою жизнь часами, а я служу вам истечением срока. Я быстрая, когда я худой, и медленная, когда я толстая. Ветер — мой враг.

Ответ:

Свеча.

12. Вы видите лодку, заполненную людьми. Он не затонул, но когда вы посмотрите еще раз, вы не увидите ни одного человека на лодке. Почему?

Ответ:

Все были женаты.

Короткие невозможные загадки

Короткие и сложные загадки непросты, потому что в них не так много текста, чтобы помочь вам найти решение. Они побуждают к логическому мышлению, но вы должны сосредоточиться на получении правильного ответа. Итак, каковы лучшие короткие, неразрешимые загадки и ответы на них?

13.Какое английское слово состоит из трех последовательных двойных букв?

Ответ:

Бухгалтер.

14. Что имеет глаз, но не видит?

Ответ:

Игла.

(У иглы есть «ушко», в котором проходит нить, но она не видит им)

Читайте также

Самые острые любовные цитаты всех времен

ЧИТАЙТЕ ТАКЖЕ: Цитаты Пророка о Рамадане Мухаммад и Коран

15. Что летает без крыльев?

Ответ:

Время.

16. Каков конец радуги?

Ответ:

Буква W.

17. Переверни меня на бок, и я все. Разрежь меня пополам, и я ничто. Что я?

Ответ:

Число 8.

18. Как разместить 10 лошадей в 9 стойлах?

Ответ:

[t] [e] [n] [h] [o] [r] [s] [e] [s]

19. Что вы можете держать в правой руке, но не можете оставил?

Ответ:

Правый локоть.

Сложные загадки, которые заставляют вас выглядеть тупым

Изображение: pixabay.com, (изменено автором)
Источник: UGC

Если вы хотите дать своему мозгу тренировку, то решите самые сложные загадки, которые заставят вас выглядеть глупо. В конце концов, вы получите лучшую головоломку для себя и, вероятно, получите решение после нескольких часов работы над своим мозгом.

ЧИТАЙТЕ ТАКЖЕ: Лучшие цитаты и пожелания Рамадана

Читайте также

Лучшие веселые цитаты о жизни в целом

20.Вы выходите из лабиринта, а перед вами три двери. Дверь слева ведет к бушующему аду, а центральная — к смертоносному убийце. Дверь справа ведет к льву, который не ел три месяца. Какую дверь выбрать?

Ответ:

Лев умер бы, если бы не ел в течение трех месяцев.

21. Умер старик, оставив двух сыновей. В своем завещании он приказывает своим сыновьям участвовать в гонках со своими лошадьми, и тот, у кого лошадь будет медленнее, получит его наследство.Два сына участвуют в гонке, но, поскольку они оба сдерживают своих лошадей, они идут к одному мудрецу и спрашивают его, что им делать. После этого братья снова участвуют в гонке — на этот раз полным ходом. Что сказал им мудрец?

Ответ:

После того, как они поменяют лошадь, любой, кто выиграет скачку, получит наследство, поскольку технически он все еще владеет проигравшей лошадью.

Прочтите также

25 лучших диких цитат для девочек

22. Я знаю тысячу лиц и считаю хвостатых голов, ярких на глазах у многих умерших, хорошо владеющих могучей силой, у кого скромный рост .Массы падают на колени, чтобы не видеть мою единственную сторону.

Ответ:

Монета.

23. Есть три сундука, в каждом по 100 монет. В одном сундуке 100 золотых монет, в другом 100 серебряных монет, а в третьем поровну 50 золотых и 50 серебряных монет. Каждый сундук промаркирован, но не все. Вам разрешено выбрать одну монету только из одного из ящиков, и после этого вы должны правильно идентифицировать каждый из трех сундуков. Что вы должны сделать?

Ответ:

Возьмите монету из сундука с надписью 50/50.Если вы получите золотую монету, вы узнаете, что в сундуке есть только золотые монеты. Следовательно, помеченное серебром должно быть сундуком 50/50, а помеченное золотом — серебряным сундуком.

Читайте также

20 самых известных цитат Сократа

24. Вы должны держать меня как можно более прямолинейным, но очень немногие это делают. Большую часть времени я слегка согнут или искривлен. Ваша печаль обычно заставляет меня сгибаться еще больше, но не сгибайте меня слишком долго, иначе я никогда не смогу полностью выпрямиться снова.Что я?

Ответ:

Ваша поза.

ЧИТАЙТЕ ТАКЖЕ: Пасхальные послания 2020 для друзей и семьи

Изображение: pixabay.com, (изменено автором)
Источник: UGC

25. Солнце их поджигает, рука ломает их, нога наступает на них, а их вкус во рту. Кто они такие?

Ответ:

Виноград.

26. Когда на меня брызгает жидкость, она не просачивается. Когда я много двигаюсь, я изрыгаю жидкость. Когда меня бьют, я меняю цвет, и цвета у меня довольно разные.То, что я покрываю, довольно сложно, но я элементарно сгибаюсь. Что я?

Ответ:

Скин.

27. Я двигаюсь очень медленно с незаметной скоростью, хотя я не тороплюсь, я никогда не опаздываю. Я сопровождаю жизнь и переживаю прошлую кончину; Многие женщины смотрят на меня с уважением. Что я?

Читайте также

11 вещей, которые меняют женщины после того, как они говорят, что я делаю, что раздражает мужчин

Ответ:

Я — твои волосы.

28.Врач и водитель автобуса влюблены в одну и ту же женщину, в привлекательную девушку по имени Сара. Водителю автобуса пришлось отправиться в длительную поездку на автобусе, которая продлилась неделю. Перед отъездом он дал Саре семь яблок. Почему?

Ответ:

Яблоко в день отпугнет врача.

29. Мойщик окон моет окно на 25-м этаже небоскреба, как вдруг он шлепается и падает. У него нет защитного снаряжения и ничего, что могло бы смягчить его падение, и тем не менее он не пострадал.Как это может быть?

Ответ:

Он был внутри мыть окна.

ЧИТАЙТЕ ТАКЖЕ: Веселые пасхальные цитаты 2020

30. Какую высоту вам придется сосчитать, прежде чем использовать букву А в английском написании целого числа?

Ответ:

Одна тысяча.

Изображение: pexels.com, (изменено автором)
Источник: UGC

Загадки невозможно разгадать, и вы можете бросить своему мозгу идеальный вызов в любой момент, когда почувствуете, что готовы к этому.А кто не любит головоломки? Если вы думаете, что уже являетесь профессионалом в разгадывании сложных словесных головоломок, дайте себе идеальный тест, разгадывая почти невозможные загадки. Они подарят вам идеальный кайф.

Подпишитесь, чтобы смотреть новые видео

ЧИТАЙТЕ ТАКЖЕ:

  • Короткие пасхальные цитаты из Библии
  • 10 мощных молитв о защите от болезней и болезней
  • 100+ лучших статусных цитат в WhatsApp

Источник : Туко.co.ke

20+ запутанных загадок с ответами, которые являются настоящими царапинами для головы от Kidadl

Хитрые загадки и головоломки — это те, которые заставляют ваш мозг думать об одном конкретном предмете, тем самым повышая уровень концентрации и IQ.

По-настоящему запутанные загадки, которые сложно разгадать, могут заставить вас думать весь день, тем самым помогая вашему мозгу сосредоточиться. Это отличное упражнение для вашего мозга, если вы дадите себе или своему ребенку время в течение дня, когда вы сидите и придумываете ответ на набор очень запутанных загадок.

Хитрые загадки действительно могут помочь ребенку стать умнее и острее, улучшить его навыки решения проблем и помочь им думать, что, как оказалось, станет отличным жизненным навыком, когда он вырастет. Вы можете составить список сложных загадок с ответами и задавать их через день. Это не только отличный способ провести время с ребенком, но и поможет получить тяжелую тренировку как для вас, так и для ума вашего ребенка. Сосредоточение внимания на чем-то одном, в частности, помогает повысить уровень концентрации, а также является отличным усилителем памяти.Если вы ищете одни из лучших загадок для себя и своего ребенка и любите думать над ответом на сложную загадку, этот список обязательно заставит вас ломать голову. Если вам также нравится такой контент, вы можете ознакомиться с другими статьями, такими как Загадки Детективов и Загадки Сфинкса, чтобы получить лучшие загадки!

Запутанные загадки с простыми ответами

Вы ищете непонятные вопросы с ответами? Вот список очень хороших загадок, которые вам обязательно понравятся.

1. Замков нет, но ключи есть. У меня нет места, но есть место. Вы не можете выйти на улицу, но можете войти. Что я? Ответ: клавиатура.

2. Что вы обычно получаете один раз в минуту, два раза в мгновение, но никогда не приходит за тысячу лет? Ответ: Буква «М».

3. Какое словарное слово написано с ошибками? Ответ: Слово «неправильно».

4. На какой один вопрос в разное время будут даны разные ответы, но при этом он будет правильным? Ответ: «Сколько времени?»

5.Два сына и два отца выходят на рыбалку. Они ловят по одной рыбе, но приносят домой только три. Почему? Ответ: Это были отец, дед и сын.

6. Есть три типа печей: кирпичная печь, стеклянная печь и дровяная печь, но у вас есть только одна спичка. Какой из них нужно зажечь первым? Ответ: Матч.

7. Что начинается с Т, заканчивается буквой Т, но также имеет Т внутри? Ответ: Чайник

8. У меня леса без деревьев, города без домов, вода без волн.Что я? Ответ: Карта

9. Хелен, Джордж и Эллен пьют кофе, а Макс, Барб и Дэйв пьют 7up. Что пьет Элизабет; кофе или газировка? Ответ: Элизабет пьет кофе. (В ее имени буква E встречается дважды, как и в именах других любителей кофе.)

10. Что вы оставляете себе после того, как дадите это кому-то? Ответ: Ваше слово.

11. У меня нет рта, но я всегда отвечаю, когда вы говорите со мной. Что я? Ответ: Эхо.

12. Сколько месяцев в году состоит из 28 дней? Ответ: Все месяцы.

13. Кто может бегать, но никогда не ходит и у кого есть рот? Ответ: Река.

14. У чего есть шея, но нет головы? Ответ: Рубашка.

15. Кто имеет лицо и руки, но ничего не может улыбнуться или удержать? Ответ: Часы.

16. Что принадлежит вам, но другие чаще используют это? Ответ: Ваше имя.

17. Что единственное в мире идеально? Ответ: Слово «идеальный».«

18. Какие только две вещи нельзя есть на завтрак? Ответ: Обед и ужин.

19. Какое слово становится короче, когда к нему добавляются 2 буквы? Ответ: Слово« короткое ».

20. Что такое столица Франции? Ответ: буква F.

Сложные и забавные загадки и ответы

Ищете смешные и запутанные загадки с простыми ответами, чтобы взломать своих детей? В этом списке есть несколько отличных вопросов, чтобы сбить с толку людей и поможет разгадывать лучшие загадки!

21.Ковбой едет в город в понедельник и остается там на три дня, а затем уезжает в понедельник. Как? Ответ: Понедельник — это имя его лошади.

22. Что может путешествовать по миру, но не может покинуть угол? Ответ: штамп.

24. Что меньше весит, когда становится больше? Ответ: Попкорн.

25. Что может заполнить всю комнату, но не занимать места? Ответ: Свет.

Самая сложная загадка в мире

Вот самая сложная загадка в мире, которую не смогли разгадать 97% выпускников Гарварда.Однако 84% воспитанников детского сада смогли решить ее за 6 минут и даже меньше.

26. Я могу заставить белых медведей побелеть, и я даже могу заставить вас плакать. Я заставляю парней сходить в туалет, а девушек — причесываться. Я могу заставить известных людей выглядеть глупо, а нормальных людей — знаменитыми. Я могу сделать блины коричневыми и сделать бокал шампанского пузырями. Если меня сжимают, я лопну, и если ты посмотришь на меня, ты тоже лопнешь. Отгадаешь загадку? Ответ: Нет (последнее предложение — «Сможете ли вы отгадать загадку?»)

Самая сложная головоломка в мире

Вот самая сложная головоломка в мире, цитируемая прямо из уст Ричарда Смолляна, математика и был назван «бесспорным мастером логических головоломок».«Эта головоломка считается самой сложной логической головоломкой, и ее еще предстоит решить. Однако вы можете попытать счастья с ней, если готовы принять хороший вызов.

27. Есть три бога с именами A, B и В. Они называются в некотором порядке Истинным, Ложным и Случайным образом. Истинный, как известно, всегда говорит правду. Ложь, как известно, всегда говорит ложно. Однако, если Случайный говорит ложно или правдиво, это совершенно случайно. узнайте личности A, B и C, задав всего три вопроса типа «да» или «нет».Каждый из вопросов должен быть задан только одному богу. Боги знают английский, но могут ответить на все ваши вопросы на своем родном языке. На их языке «да» и «нет» переводятся как «да» и «джа» в некотором порядке, но вы не можете узнать, какое слово что означает. Как бы вы отождествили каждого бога с каждым вопросом?

Здесь, в Kidadl, мы собрали множество замечательных загадок для всей семьи, чтобы каждый мог насладиться им! Если вам понравились наши предложения по разгадыванию загадок, почему бы не взглянуть на «Страшные загадки» или «Загадки любви».

Самые смешные загадки с ответами в 2021 году

(прокрутите вниз или выберите другую категорию)

Что такое забавные загадки?

Загадка — это вопрос, который требует от человека, которому задают загадку, использовать свой интеллект и навыки мышления, чтобы ответить на него. Обычно, чтобы ответить на загадку, человек должен мыслить нестандартно, чтобы получить ответ. Забавная загадка похожа на обычную загадку, но она заставит вас смеяться! Вот наша коллекция.

Загадка: Какие две вещи нельзя есть на завтрак?

Ответ: Обед и ужин.

Загадка: У чего есть лицо и две руки, но нет рук и ног?

Ответ: Часы.

Загадка: Что на вкус лучше, чем запах?

Ответ: Язык.

Загадка: В какой комнате нет дверей и окон?

Ответ: Гриб.

Загадка: Что общего у Александра Великого и Винни-Пуха?

Ответ: То же отчество.

Загадка: В какой месяц люди меньше всего спят?

Ответ: Февраль — самый короткий месяц.

Загадка: Что такое оранжевый и звучит как попугай?

Ответ: Морковь.

Загадка: Что поднимается и никогда не опускается?

Ответ: Ваш возраст.

Загадка: Какое слово становится короче, если к нему добавить две буквы?

Ответ: Короткий

Загадка: Двое детей рождаются в один день от одной матери, но они не близнецы. Как такое возможно?

Ответ: Они тройняшки!

Загадка: У матери Джонни было трое детей.Первого ребенка назвали Эйприл, второго ребенка назвали Мэй. Как звали третьего ребенка?

Ответ: Джонни, конечно.

Загадка: Перед Mt. Был открыт Эверест, какая гора была самой высокой в ​​мире?

Ответ: Mt. Эверест; это просто еще не было обнаружено.

Загадка: Сколько грязи в яме размером два на три на четыре фута?

Ответ: Нет грязи, потому что это дыра.

Загадка: Что может путешествовать по земному шару, но все время остается в углу?

Ответ: Штамп.

Загадка: У чего четыре колеса и мухи?

Ответ: Мусоровоз.

Загадка: Чем больше высыхает, тем влажнее становится?

Ответ: Полотенце.

Загадка: Какое слово в словаре написано неправильно?

Ответ: Неправильно

Загадка: Вы не можете сохранить это, пока не отдадите.Что это?

Ответ: Обещание.

Загадка: Что растет, когда ест, и умирает, когда пьет?

Ответ: Огонь.

Загадка: Что все время проводит на полу, но никогда не пачкается?

Ответ: Твоя тень.

Загадка: Какое животное после смерти оборачивается примерно 200 раз вокруг своей оси?

Ответ: Жареный цыпленок.

Загадка: Сколько месяцев в году 28 дней?

Ответ: Все.

Загадка: Человек выходит под проливным дождем, и ничто не может его защитить. Его волосы не промокают. Как он это делает?

Ответ: Он лысый.

Загадка: В какой части курицы больше всего перьев?

Ответ: Снаружи.

Загадка: Что поднимается и опускается, но не движется?

Ответ: Лестница.

Загадка: Десять женщин пытались уместиться под маленьким зонтиком, но ни одна из них не промокла. Как они это делают?

Ответ: Дождя не было!

Загадка: Что звучит как чихание и сделано из кожи?

Ответ: Обувь

Загадка: Что всегда ложится спать в обуви?

Ответ: Лошадь

Загадка: Что внизу вверху?

Ответ: Ваши ноги.

Загадка: Если двое — компания, а трое — толпа, то что такое четыре и пять?

Ответ: Девять конечно.

Загадка: Джим шести футов ростом, он работает в мясной лавке и носит туфли десятого размера.

Сколько он весит?

Ответ: Мясо

Загадка: что начинается с Е, заканчивается на Е и содержит только одну букву?

Ответ: конверт

Загадка: Что бежит, но не ходит?

Ответ: тап.

Загадка: Что можно подарить мужчине без волос, с которым он никогда не расстанется?

Ответ: Расческа.

Загадка: Какой любимый овощ цыплят?

Ответ: Баклажан.

Загадка: Какие овощи нужны сантехнику?

Ответ: лук-порей.

Загадка: Если Y.E.S пишет «да», что означает E.Y.E.S?

(лучше всего работает, когда говорят другу, а не читают)

Загадка: Как далеко медведь может зайти в лес?

Ответ: На полпути.

Загадка: Какие 5-буквенные слова в словаре можно произносить одинаково, даже если из него удалены 4 буквы?

Ответ: Очередь

Загадка: Что вы можете легко держать в правой руке, но никогда в левой?

Ответ: Ваш левый локоть.

Загадка: две монеты в сумме дают 30 центов, и одна из них не никель. Кто они такие?

Четверть и пятак. Квартал — это не монета.

Загадка: Вперед я тяжелый, назад нет. Что я?

Ответ: Тонна

Загадка: Что такое красный и пахнет синей краской?

Ответ: Красная краска.

Загадка: какое слово в словаре начинается и заканчивается буквой E, но имеет только одну букву?

Ответ: Конверт.

Загадка: У чего есть шея, но нет головы?

Ответ: Бутылка.

Загадка: Почему этому человеку пришлось закопать свой факел?

Ответ: Потому что батарейки сели.

Загадка: что начинается с буквы P, заканчивается буквой E и содержит более тысячи букв?

Ответ: Почтовое отделение.

Загадка: Почему женщину, живущую в Новой Зеландии, никогда не похоронят в Австралии?

Ответ: Потому что она живая.

Загадка: 3 джентльмена выходят на рыбалку в лодке, когда волна-убийца сбивает их всех за борт и под водой. Только у меня волосы намокли. Почему?

Ответ: 2 из них были лысыми.

Загадка: Если бы все решили покупать только розовые машины, что бы у нас было?

Ответ: Гвоздика розовая.

Загадка: Когда дверь перестает быть дверью?

Ответ: Когда он приоткрыт.

Загадка: В чем главное отличие человека, торгующего часами, от тюремного надзирателя?

Ответ: Один продает часы, другой часы сотовые.

Загадка: Маленький Джимми бросает теннисный мяч так сильно и далеко, как только может. мяч возвращается прямо к нему. Как это произошло?

Ответ: Джимми бросает мяч прямо вверх.

Загадка: Что такое невероятно хрупкое, что, просто произнеся его имя, оно сломается?

Ответ: Тишина.

Загадка: Ходите по живым, они даже не бормочут. Ходите по мертвым, а они бормочут и ворчат. Кто они такие?

Ответ: оставляет

Загадка: 2 парня в лодке с 3-мя сигаретами. К сожалению, у них нет спичек, как им удается прикурить сигареты?

Ответ: Они выбрасывают за борт 1 сигарету, и от этого лодка становится немного легче.

Загадка: есть 10 детей и миска с 10 апельсинами. Как разделить апельсины так, чтобы каждый ребенок получил по апельсину, а один остался в миске?

Ответ: Вы даете 9 детям по апельсину, а десятому ребенку — миску с апельсином.

Загадка: Два мальчика родились от одной матери в один и тот же день, в одно и то же время, в один и тот же месяц и год, но они не близнецы.Как это возможно?

Ответ: Они являются частью набора троек.

Загадка: Молодой человек учится водить машину. Он едет не в ту сторону по улице с односторонним движением и проезжает мимо полицейского. Полицейский видит его, но ничего не делает. Почему?

Ответ: Молодой человек шел.

Загадка: что начинается с Т, заканчивается Т и полно Т?

Ответ: Чайник.

Загадка: Я невесом, но положи меня в ведро, и я сделаю его легче. Что я?

Ответ: Дырка.

Загадка: У чего много сердец, но нет других органов?

Ответ: Колода карт.

Загадка: Что иногда зависает после нагрева?

Ответ: Компьютер.

Загадка: сорванный с плода, горький, как корень, жгучий и жирный, или даже холодный при продаже. Что я?

Ответ: Кофе в зернах.

Leave a Reply